1.5.0.0.1.a.1 G10 ALL Notification to the NTSB is required when there has been substantial damage A. which requires repairs to landing gear. B. to an engine caused by engine failure in flight. C. which adversely affects structural strength or flight characteristics. C 1.5.0.0.2.a.1 G11 ALL NTSB Part 830 requires an immediate notification as a result of which incident? A. Engine failure for any reason during flight. B. Damage to the landing gear as a result of a hard landing. C. Any required flight crewmember being unable to perform flight duties because of illness. C 1.5.0.0.3.a.1 G11 ALL Which incident would require that the nearest NTSB field office be notified immediately? A. In-flight fire. B. Ground fire resulting in fire equipment dispatch. C. Fire of the primary aircraft while hangered which results in damage to other property of more than $50,000. A 1.5.0.0.4.a.1 G11 ALL While taxiing for takeoff, a small fire burned the insulation from a transceiver wire. What action would be required to comply with NTSB Part 830? A. No notification or report is required. B. A report must be filed with the avionics inspector at the nearest FAA field office within 48 hours. C. An immediate notification must be filed by the operator of the aircraft with the nearest NTSB field office. A 1.5.0.0.5.a.1 G11 ALL During flight a fire which was extinguished burned the insulation from a transceiver wire. What action is required by regulations? A. No notification or report is required. B. Report must be filed with the avionics inspector at the nearest FAA field office within 48 hours. C. An immediate notification by the operator of the aircraft to the nearest NTSB field office. C 1.5.0.0.6.a.1 G11 ALL When should notification of an aircraft accident be made to the NTSB if there was substantial damage and no injuries? A. Immediately. B. Within 10 days. C. Within 30 days. A 1.5.0.0.7.a.1 G13 ALL The operator of an aircraft that has been involved in an incident is required to submit a report to the nearest field office of the NTSB A. within 7 days. B. within 10 days. C. only if requested to do so. C 1.5.0.0.8.a.1 G13 ALL Within how many days of an accident is an accident report required to be filed with the nearest NTSB field office? A. 2 days. B. 7 days. C. 10 days. C 1.5.0.0.9.a.1 A01 ALL What designated airspace associated with an airport becomes inactive when the control tower at that airport is not in operation? A. Class D, which then becomes Class C. B. Class D, which then becomes Class E. C. Class B. B 1.5.0.1.0.a.1 A01 ALL Regulations which refer to commercial operators relate to that person who A. is the owner of a small scheduled airline. B. for compensation or hire, engages in the carriage by aircraft in air commerce of persons or property, as an air carrier. C. for compensation or hire, engages in the carriage by aircraft in air commerce of persons or property, other than as an air carrier. C 1.5.0.1.1.a.1 A01 ALL Regulations which refer to operate relate to that person who A. acts as pilot in command of the aircraft. B. is the sole manipulator of the aircraft controls. C. causes the aircraft to be used or authorizes its use. C 1.5.0.1.2.a.1 A01 ALL Regulations which refer to the operational control of a flight are in relation to A. the specific duties of any required crewmember. B. acting as the sole manipulator of the aircraft controls. C. exercising authority over initiating, conducting, or terminating a flight. C 1.5.0.1.8.a.1 A20 ALL Commercial pilots are required to have a current and appropriate pilot certificate in their personal possession when A. piloting for hire only. B. carrying passengers only. C. acting as pilot in command. C 1.5.0.2.0.a.1 A20 ALL Does a commercial pilot certificate have a specific expiration date? A. No, it is issued without an expiration date. B. Yes, it expires at the end of the 24th month after the month in which it was issued. C. No, but commercial privileges expire if a flight review is not satisfactorily completed each 12 months. A 1.5.0.2.1.a.1 A20 AIR,RTC,LTA A second-class medical certificate issued to a commercial pilot on April 10, this year, permits the pilot to exercise which of the following privileges? A. Commercial pilot privileges through April 30, next year. B. Commercial pilot privileges through April 10, 2 years later. C. Private pilot privileges through, but not after, March 31, next year. A 1.5.0.2.2.a.1 A20 AIR,RTC When is the pilot in command required to hold a category and class rating appropriate to the aircraft being flown? A. All solo flights. B. Flight tests given by the FAA. C. Flights for compensation or hire. C 1.5.0.2.5.a.1 A20 AIR,RTC What flight time may a pilot log as second in command? A. All flight time while acting as second in command in aircraft requiring more than one pilot. B. Only that flight time during which the second in command is the sole manipulator of the controls. C. All flight time while acting as second in command regardless of aircraft crew requirements. A 1.5.0.2.6.a.1 A20 ALL What flight time must be shown, in a reliable record, by a pilot exercising the privileges of a commercial certificate? A. Flight time showing aeronautical training and experience to meet requirements for a certificate or rating. B. All flight time flown for compensation or hire. C. Only flight time for compensation or hire with passengers aboard which is necessary to meet the recent flight experience requirements. A 1.5.0.2.7.a.1 A20 ALL If a pilot does not meet the recency of experience requirements for night flight and official sunset is 1800 CST, the latest time passengers should be carried is A. 1759 CST. B. 1829 CST. C. 1859 CST. C 1.5.0.2.8.a.1 A20 ALL Prior to carrying passengers at night, the pilot in command must have accomplished the required takeoffs and landings in A. any category aircraft. B. the same category and class of aircraft to be used. C. the same category, class, and type of aircraft to be used. B 1.5.0.2.9.a.1 A20 RTC To act as pilot in command of a gyroplane carrying passengers, what must the pilot do in that gyroplane to meet recent daytime flight experience requirements? A. Make 9 takeoffs and landings within the preceding 30 days. B. Make 3 takeoffs and landings to a full stop within the preceding 90 days. C. Make 3 takeoffs and landings within the preceding 90 days. C 1.5.0.3.1.a.1 A20 ALL To act as pilot in command of an aircraft under FAR Part 91, a commercial pilot must have satisfactorily accomplished a flight review or completed a proficiency check within the preceding A. 6 months. B. 12 months. C. 24 months. C 1.5.0.3.2.a.1 A20 ALL Pilots who change their permanent mailing address and fail to notify the FAA Airmen Certification Branch of this change, are entitled to exercise the privileges of their pilot certificate for a period of A. 30 days. B. 60 days. C. 90 days. A 1.5.0.4.3.a.1 A60 ALL Excluding Hawaii, the vertical limits of the Federal Low Altitude airways extend from A. 700 feet AGL up to, but not including, 14,500 feet MSL. B. 1,200 feet AGL up to, but not including, 18,000 feet MSL. C. 1,200 feet AGL up to, but not including, 14,500 feet MSL. B 1.5.0.4.4.a.1 A60 ALL One of the major differences between Class D airspace and Class E airspace is that Class D airspace A. is located at tower-controlled airports and Class E airspace is at uncontrolled airports. B. always begins at 700 feet AGL while Class E always begins at 1,200 feet above the surface. C. begins at the surface, while Class E always begins at an altitude of 700 feet or 1,200 feet above the surface. A 1.5.0.4.5.a.1 A60 ALL The Continental Control Area A. does not exist anymore. B. extends upward from 10,000 feet MSL. C. extends upward from 14,500 feet MSL. A 1.5.0.4.6.a.1 A60 ALL Within the contiguous U.S., the vertical limit of Class D airspace normally extends from the surface upward to A. infinity. B. but not including the base of Class A airspace. C. 2,500 feet AGL or indicated within a square depicted within that airspace on aeronautical charts. C 1.5.0.4.7.a.1 A60 ALL Which is true regarding Class E airspace? A. The basic VFR minimums are greater than those associated Class D airspace. B. Class E airspace may start at the surface, but usually begins at an altitude of 700 feet or 1,200 feet above the surface. C. Class E airspace begins at the surface and extend upward to Flight Level 600. B 1.5.0.4.9.a.1 B07 ALL The required preflight action relative to alternatives available, if the planned flight cannot be completed, is applicable to A. IFR flights only. B. any flight not in the vicinity of an airport. C. any flight conducted for hire or compensation. B 1.5.0.5.0.a.1 B07 ALL Before beginning any flight under IFR, the pilot in command must become familiar with all available information concerning that flight. In addition, the pilot must A. be familiar with all instrument approaches at the destination airport. B. list an alternate airport on the flight plan and confirm adequate takeoff and landing performance at the destination airport. C. be familiar with the runway lengths at airports of intended use, and the alternatives available if the flight cannot be completed. C 1.5.0.5.1.a.1 B07 AIR,RTC,LTA Required flight crewmembers' seatbelts must be fastened A. only during takeoff and landing. B. while the crewmembers are at their stations. C. only during takeoff and landing when passengers are aboard the aircraft. B 1.5.0.5.2.a.1 B07 ALL The use of seatbelts, with certain exceptions, during takeoffs and landings is A. required for all occupants. B. required during commercial operations only. C. a good operating practice, but not required by regulations. A 1.5.0.5.6.a.1 B07 ALL Portable electronic devices which may cause interference with the navigation or communication system may not be operated on aircraft being flown A. along Federal airways. B. within the U.S. C. in commercial operations. C 1.5.0.5.8.a.1 B07 RTC To begin a flight in a rotorcraft under VFR, there must be enough fuel to fly to the first point of intended landing and, assuming normal cruise speed, to fly thereafter for at least A. 20 minutes. B. 30 minutes. C. 45 minutes. A 1.5.0.5.9.a.1 B07 AIR,RTC,LTA If weather conditions are such that it is required to designate an alternate airport on your IFR flight plan, you should plan to carry enough fuel to arrive at the first airport of intended landing, fly from that airport to the alternate airport, and fly thereafter for A. 30 minutes at slow cruising speed. B. 45 minutes at normal cruising speed. C. 1 hour at normal cruising speed. B 1.5.0.6.0.a.1 B07 AIR,RTC A coded transponder equipped with altitude reporting equipment is required for A. Class A, Class B, and Class C airspace areas. B. all airspace of the 48 contiguous U.S. and the District of Columbia at and above 10,000 feet MSL (including airspace at and below 2,500 feet above the surface). C. both answer A and B. A 1.5.0.6.1.a.1 B07 AIR,RTC In the contiguous U.S., excluding the airspace at and below 2,500 feet AGL, an operable coded transponder equipped with Mode C capability is required in all airspace above A. 10,000 feet MSL. B. 12,500 feet MSL. C. 14,500 feet MSL. A 1.5.0.6.2.a.1 B07 AIR,LTA,RTC What is the maximum tolerance (+ or -) allowed for an operational VOR equipment check when using a VOT? A. 4°. B. 6°. C. 8°. A 1.5.0.6.3.a.1 B07 ALL In accordance with FAR Part 91, supplemental oxygen must be used by the required minimum flightcrew for that time exceeding 30 minutes while at cabin pressure altitudes of A. 10,500 feet MSL up to and including 12,500 feet MSL. B. 12,000 feet MSL up to and including 18,000 feet MSL. C. 12,500 feet MSL up to and including 14,000 feet MSL. C 1.5.0.6.4.a.1 B07 ALL What are the oxygen requirements when operating above 15,000 feet MSL? A. Oxygen must be available for the flightcrew. B. Oxygen is not required at any altitude in a free balloon. C. The flightcrew must use and passengers must be provided oxygen. C 1.5.0.6.5.a.1 B07 AIR,RTC,GLI Which is required equipment for powered aircraft during VFR night flights? A. Anticollision light system. B. Gyroscopic direction indicator. C. Gyroscopic bank-and-pitch indicator. A 1.5.0.6.8.a.1 B07 RTC Which is true with respect to operating limitations of a ``restricted'' category helicopter? A. A restricted category helicopter is limited to an operating radius of 25 miles from its home base. B. A pilot of a restricted category helicopter is required to hold a commercial pilot certificate. C. No person may operate a restricted category helicopter carrying property or passengers for compensation or hire. C 1.5.0.7.0.a.1 B07 AIR,RTC The maximum cumulative time that an emergency locator transmitter may be operated before the rechargeable battery must be recharged is A. 30 minutes. B. 45 minutes. C. 60 minutes. C 1.5.0.7.2.a.1 B08 RTC What transponder equipment is required for helicopter operations within Class B airspace? A transponder A. with 4096 code and Mode C capability. B. is required for helicopter operations when visibility is less than 1 mile. C. with 4096 code capability is required except when operating at or below 1,000 feet AGL under the terms of a letter of agreement. A 1.5.0.7.3.a.1 B08 ALL Which is true with respect to formation flights? Formation flights are A. authorized when carrying passengers for hire with prior arrangement with the pilot in command of each aircraft in the formation. B. not authorized when visibilities are less than 3 SM. C. not authorized when carrying passengers for hire. C 1.5.0.7.4.a.1 B08 AIR,RTC While in flight a helicopter and an airplane are converging at a 90° angle, and the helicopter is located to the right of the airplane. Which aircraft has the right-of-way, and why? A. The helicopter, because it is to the right of the airplane. B. The helicopter, because helicopters have the right-of-way over airplanes. C. The airplane, because airplanes have the right-of-way over helicopters. A 1.5.0.7.5.a.1 B08 ALL Two aircraft of the same category are approaching an airport for the purpose of landing. The right-of-way belongs to the aircraft A. at the higher altitude. B. at the lower altitude, but the pilot shall not take advantage of this rule to cut in front of or to overtake the other aircraft. C. that is more maneuverable, and that aircraft may, with caution, move in front of or overtake the other aircraft. B 1.5.0.7.6.a.1 B08 AIR,RTC Airplane A is overtaking airplane B. Which airplane has the right-of-way? A. Airplane A; the pilot should alter course to the right to pass. B. Airplane B; the pilot should expect to be passed on the right. C. Airplane B; the pilot should expect to be passed on the left. B 1.5.0.8.0.a.1 B08 ALL If not equipped with required position lights, an aircraft must terminate flight A. at sunset. B. 30 minutes after sunset. C. 1 hour after sunset. A 1.5.0.8.2.a.1 B08 ALL Which is true regarding VFR operations in Class B airspace? A. Area navigation equipment is required. B. Flight under VFR is not authorized unless the pilot in command is instrument rated. C. Solo student pilot operations are allowed if certain conditions are satisfied. C 1.5.0.8.3.a.1 B08 ALL The minimum flight visibility for VFR flight increases to 5 miles beginning at an altitude of A. 14,500 feet MSL. B. 10,000 feet MSL if above 1,200 feet AGL. C. 10,000 feet MSL regardless of height above ground. B 1.5.0.8.5.a.1 B08 ALL What is the minimum flight visibility and proximity to cloud requirements for VFR flight, at 6,500 feet MSL, in Class C, D, and E airspace? A. 1 mile visibility; clear of clouds. B. 3 miles visibility; 1,000 feet above and 500 feet below. C. 5 miles visibility; 1,000 feet above and 1,000 feet below. B 1.5.0.8.6.a.1 B08 RTC Which minimum flight visibility and distance from clouds is required for a day VFR helicopter flight in Class G airspace at 3,500 feet MSL over terrain with an elevation of 1,900 feet MSL? A. Visibility-3 miles; distance from clouds-1,000 feet below, 1,000 feet above, and 1 mile horizontally. B. Visibility-3 miles; distance from clouds-500 feet below, 1,000 feet above, and 2,000 feet horizontally. C. Visibility-1 mile; distance from clouds-500 feet below, 1,000 feet above, and 2,000 feet horizontally. C 1.5.0.8.7.a.1 B08 RTC Basic VFR weather minimums require at least what visibility for operating a helicopter within Class D airspace? A. 1 mile. B. 2 miles. C. 3 miles. C 1.5.0.9.1.a.1 B08 AIR,RTC,LTA VFR cruising altitudes are required to be maintained when flying A. at 3,000 feet or more AGL; based on true course. B. more than 3,000 feet AGL; based on magnetic course. C. at 3,000 feet or more above MSL; based on magnetic heading. B 1.5.0.9.2.a.1 B08 ALL Except when necessary for takeoff or landing or unless otherwise authorized by the Administrator, the minimum altitude for IFR flight is A. 3,000 feet over all terrain. B. 3,000 feet over designated mountainous terrain; 2,000 feet over terrain elsewhere. C. 2,000 feet above the highest obstacle over designated mountainous terrain; 1,000 feet above the highest obstacle over terrain elsewhere. C 1.5.0.9.3.a.1 B13 ALL Who is primarily responsible for maintaining an aircraft in an airworthy condition? A. The lead mechanic responsible for that aircraft. B. Pilot in command. C. Operator or owner of the aircraft. C 1.5.0.9.4.a.1 B13 ALL Assuring compliance with an Airworthiness Directive is the responsibility of the A. pilot in command and the FAA certificated mechanic assigned to that aircraft. B. pilot in command of that aircraft. C. owner or operator of that aircraft. C 1.5.0.9.5.a.1 B13 ALL After an annual inspection has been completed and the aircraft has been returned to service, an appropriate notation should be made A. on the airworthiness certificate. B. in the aircraft maintenance records. C. in the FAA-approved flight manual. B 1.5.0.9.6.a.1 B13 ALL The validity of the airworthiness certificate is maintained by A. performance of an annual inspection. B. performance of an annual inspection and a 100-hour inspection prior to their expiration date. C. an appropriate return to service statement in the aircraft maintenance records upon the completion of required inspections and maintenance. C 1.5.0.9.7.a.1 B13 ALL If an aircraft's operation in flight was substantially affected by an alteration or repair, the aircraft documents must show that it was test flown and approved for return to service by an appropriately-rated pilot prior to being operated A. by any private pilot. B. with passengers aboard. C. for compensation or hire. B 1.5.0.9.8.a.1 B13 ALL Which is correct concerning preventive maintenance, when accomplished by a pilot? A. A record of preventive maintenance is not required. B. A record of preventive maintenance must be entered in the maintenance records. C. Records of preventive maintenance must be entered in the FAA-approved flight B 1.5.0.9.9.a.1 B13 ALL An aircraft carrying passengers for hire has been on a schedule of inspection every 100 hours of time in service. Under which condition, if any, may that aircraft be operated beyond 100 hours without a new inspection? A. The aircraft may be flown for any flight as long as the time in service has not exceeded 110 hours. B. The aircraft may be dispatched for a flight of any duration as long as 100 hours has not been exceeded at the time it departs. C. The 100-hour limitation may be exceeded by not more than 10 hours if necessary to reach a place at which the inspection can be done. C 1.5.1.0.0.a.1 B13 ALL Which is true concerning required maintenance inspections? A. A 100-hour inspection may be substituted for an annual inspection. B. An annual inspection may be substituted for a 100-hour inspection. C. An annual inspection is required even if a progressive inspection system has been approved. B 1.5.1.0.1.a.1 B13 ALL An ATC transponder is not to be used unless it has been tested, inspected, and found to comply with regulations within the preceding A. 30 days. B. 12 calendar months. C. 24 calendar months. C 1.5.1.0.2.a.1 B13 ALL Aircraft maintenance records must include the current status of the A. applicable airworthiness certificate. B. life-limited parts of only the engine and airframe. C. life-limited parts of each airframe, engine, propeller, rotor, and appliance. C 1.5.1.0.3.a.1 B13 ALL Which is true relating to Airworthiness Directives (AD's) ? A. AD's are advisory in nature and are, generally, not addressed immediately. B. Noncompliance with AD's renders an aircraft unairworthy. C. Compliance with AD's is the responsibility of maintenance personnel. B 1.5.1.0.4.a.1 B13 AIR,RTC A new maintenance record being used for an aircraft engine rebuilt by the manufacturer must include previous A. operating hours of the engine. B. annual inspections performed on the engine. C. changes as required by Airworthiness Directives. C 1.5.1.0.5.a.1 B13 ALL If an ATC transponder installed in an aircraft has not been tested, inspected, and found to comply with regulations within a specified period, what is the limitation on its use? A. Its use is not permitted. B. It may be used when in Class G airspace. C. It may be used for VFR flight only. A 1.5.1.0.6.a.1 D30 AIR,RTC Which of these operations could fall under the jurisdiction of FAR Part 125? A. Operations in U.S. registered civil airplanes having a seating capacity of more than 10 but less than 20 passenger seats. B. Scheduled commercial operations (not an air carrier) using an airplane having a seating capacity of 20 or more passenger seats. C. Nonscheduled commercial operations (not an air carrier) using an airplane having a maximum payload of 6,000 pounds or more. C 1.5.1.1.6.a.1 E01 AIR,RTC FAR Part 135 applies to which operation? A. Aerial work including crop dusting and spraying. B. Carrying weekend skiers for hire to another state. C. Student instruction for hire at an approved school. B 1.5.1.1.8.a.1 E01 RTC A helicopter is being operated for hire with two passengers aboard. If the flight remains within a 25-mile radius of the departure point, the operation could be conducted, with certain stipulations, under A. FAR Part 91. B. FAR Part 97. C. FAR Part 135. A 1.5.1.1.9.a.1 E01 RTC May a helicopter be operated for hire with passengers aboard and not be subject to the rules of FAR Part 135? A. Yes, by notifying the FAA 72 hours before each flight. B. Yes, for sightseeing operations within 50 miles of the flight's origin. C. No, all flights for hire must comply with FAR Part 135. A 1.5.1.2.0.a.1 E01 RTC,LTA FAR Part 135 applies to which operation? A. Nonstop sightseeing flights that begin and end at the same airport, and are conducted within a 25 SM radius of that airport. B. Aerial operations for compensation, such as aerial photography, pipeline patrol, rescue, and crop dusting. C. Commercial operations (not an air carrier) in an aircraft with less than 20 passenger seats and a maximum payload capacity of less than 6,000 pounds. C 1.5.1.2.1.a.1 E01 ALL Under FAR Part 135 operations, who is responsible for keeping copies of the ATCO manual up to date with approved changes or additions? A. Supervising FAA district office and the certificate holder. B. Each district office employee responsible for that manual. C. Each employee of the certificate holder who is furnished a manual. C 1.5.1.2.2.a.1 E01 ALL For FAR Part 135 operations, which document(s) contain(s) procedures that explain how the pilot in command knows that the required return-to-service conditions have been met? A. Daily flight log and operation specifications. B. Certificate holder's manual. C. Mechanical deviation summary guide. B 1.5.1.2.3.a.1 E01 AIR,RTC For FAR Part 135 operations, which document specifically authorizes a person to operate an aircraft in a particular geographic area? A. Letter of authorization. B. Operations specifications. C. Air taxi operating certificate. B 1.5.1.2.4.a.1 E01 ALL An aircraft may be operated in a foreign country by an FAR Part 135 operator if authorized to do so by A. that country. B. the supervising district office. C. the FAA International Field Office in that country. A 1.5.1.2.5.a.1 E02 ALL In accordance with FAR Part 135, what period of time is the minimum flightcrew required to use supplemental oxygen while cruising at 13,500 feet MSL for 3 hours 45 minutes in an unpressurized aircraft? A. 1 hour 30 minutes. B. 2 hours 30 minutes. C. 3 hours 45 minutes. C 1.5.1.2.6.a.1 E02 AIR,RTC Which person may be carried aboard an aircraft without complying with the passenger-carrying requirements of FAR Part 135? A. A crewmember or employee of another certificate holder. B. A member of the U.S. diplomatic corps on an official courier mission. C. An individual who is necessary for the safe handling of animals on the aircraft. C 1.5.1.2.7.a.1 E02 AIR,RTC For FAR Part 135 operations, what restrictions must be observed regarding the carrying of cargo in the passenger compartment? Cargo must be A. carried directly above the seated occupants in overhead bins. B. properly secured by a seatbelt or other approved tiedown. C. separated from seated passengers by a partition capable of withstanding specified stresses. B 1.5.1.2.8.a.1 E02 AIR,RTC Under FAR Part 135, which is a requirement governing the carriage of carry-on baggage? A. Carry-on baggage must be stowed ahead of all seated occupants. B. All carry-on baggage must be restrained so that its movement is prevented during turbulence. C. Any piece of carry-on baggage, regardless of size, must be properly secured by a seatbelt or tiedown device. B 1.5.1.2.9.a.1 E02 AIR,RTC In accordance with FAR Part 135, what period of time is the minimum flightcrew required to use supplemental oxygen while cruising at 12,500 feet MSL for 1 hour 50 minutes in an unpressurized aircraft? A. 55 minutes. B. 1 hour 20 minutes. C. 1 hour 50 minutes. C 1.5.1.3.0.a.1 E02 AIR,RTC In accordance with FAR Part 135, what use of supplemental oxygen is required, if any, of a pilot when cruising at 12,500 feet MSL in an unpressurized aircraft? Supplemental oxygen is A. not required at that altitude. B. to be used during the entire flight while at that altitude. C. required for that portion of the flight that is more than 60 minutes in duration while at that altitude. B 1.5.1.3.7.a.1 E03 RTC While en route over water with passengers aboard, a helicopter is required by FAR Part 135 to A. have a visibility of, at least, during the day -1/2 mile; or at night - 1 mile. B. be operated at an altitude of 1 1/2 times the altitude needed to reach shore in the event of an engine failure. C. be equipped with flotation devices. C 1.5.1.4.1.a.1 E04 RTC Under FAR Part 135, except for takeoffs and landings, a helicopter operating VFR over a congested area is required to be operated at least A. 300 feet AGL. B. 500 feet AGL. C. 700 feet AGL. A 1.5.1.4.4.a.1 E04 RTC What is the minimum visibility requirement for FAR Part 135 helicopter VFR operations in Class G airspace at less than 1,200 feet AGL? A. Day - 1/2 mile; night - 1 mile. B. Day - 1/2 mile; night - 1/2 mile. C. Day - 1 mile; night - 1 mile. A 1.5.1.4.5.a.1 E04 RTC What is the minimum visibility requirement for FAR Part 135 helicopter VFR operations in Class D airspace? A. Day - 1/2 mile; night - 1/2 mile. B. Day - 1/2 mile; night - 1 mile. C. Day - 1 mile; night - 1 mile. B 1.5.1.4.7.a.1 E04 RTC To operate a helicopter VFR over-the-top while carrying passengers, what operating limitations, in part, are required by FAR Part 135 operations? A. Two appropriately rated pilots must be aboard; autopilot not authorized. B. Weather conditions that allow descent under VFR in the event of an engine failure. C. The helicopter must be certificated for IFR flight and the pilot in command must hold an airline transport pilot certificate. B 1.5.1.4.9.a.1 E05 RTC To conduct VFR over-the-top in a helicopter under FAR Part 135, the pilot in command is required to hold at least an A. instrument - helicopter rating. B. airline transport pilot certificate with helicopter rating. C. instrument - helicopter rating and a Class I medical certificate. A 1.5.1.5.0.a.1 E05 AIR,RTC To act as pilot in command during IFR operations under FAR Part 135, how many hours of previous instrument time in actual flight is required? At least A. 50 hours. B. 75 hours. C. 100 hours. A 2.5.1.6.8.a.1 H01 RTC For gyroplanes with constant-speed propellers, the first indication of carburetor icing is usually A. a decrease in engine RPM. B. a decrease in manifold pressure. C. engine roughness followed by a decrease in engine RPM. B 2.5.1.6.9.a.1 H02 AIR,RTC Before shutdown, while at idle, the ignition key is momentarily turned OFF. The engine continues to run with no interruption; this A. is normal because the engine is usually stopped by moving the mixture to idle cut-off. B. should not normally happen and indicates a dangerous situation. C. is an undesirable practice, but indicates that nothing is wrong. B 2.5.1.7.0.a.1 H02 AIR,RTC,LTA Leaving the carburetor heat on while taking off A. leans the mixture for more power on takeoff. B. will decrease the takeoff distance. C. will increase the ground roll. C 2.5.1.7.1.a.1 H02 AIR,RTC,LTA A way to detect a broken magneto primary grounding lead is to A. idle the engine and momentarily turn the ignition off. B. add full power, while holding the brakes, and momentarily turn off the ignition. C. run on one magneto, lean the mixture, and look for a rise in manifold pressure. A 2.5.1.7.2.a.1 H02 AIR,RTC,LTA Fouling of spark plugs is more apt to occur if the aircraft A. gains altitude with no mixture adjustment. B. descends from altitude with no mixture adjustment. C. throttle is advanced very abruptly. A 2.5.1.7.3.a.1 H02 AIR,RTC,LTA The most probable reason an engine continues to run after the ignition switch has been turned off is A. carbon deposits glowing on the spark plugs. B. a magneto ground wire is in contact with the engine casing. C. a broken magneto ground wire. C 2.5.1.7.4.a.1 H02 AIR,RTC,LTA If the ground wire between the magneto and the ignition switch becomes disconnected, the engine A. will not operate on one magneto. B. cannot be started with the switch in the BOTH position. C. could accidently start if the propeller is moved with fuel in the cylinder. C 2.5.1.7.6.a.1 H02 AIR,RTC,LTA The pilot controls the air/fuel ratio with the A. throttle B. manifold pressure C. mixture control C 2.5.1.8.5.a.1 H51 AIR,RTC,LTA Detonation may occur at high-power settings when A. the fuel mixture instantaneously ignites instead of burning progressively and evenly. B. an excessively rich fuel mixture causes an explosive gain in power. C. the fuel mixture is ignited too early by hot carbon deposits in the cylinder. A 2.5.1.8.6.a.1 H51 AIR,RTC,LTA The uncontrolled firing of the fuel/air charge in advance of normal spark ignition is known as A. instantaneous combustion. B. detonation. C. pre-ignition. C 2.5.1.8.7.a.1 H51 AIR,RTC,LTA Fuel/air ratio is the ratio between the A. volume of fuel and volume of air entering the cylinder. B. weight of fuel and weight of air entering the cylinder. C. weight of fuel and weight of air entering the carburetor. B 2.5.1.8.8.a.1 H51 AIR,RTC,LTA The mixture control can be adjusted, which A. prevents the fuel/air combination from becoming too rich at higher altitudes. B. regulates the amount of air flow through the carburetor's venturi. C. prevents the fuel/air combination from becoming lean as the airplane climbs. A 2.5.1.8.9.a.1 H51 AIR,RTC,LTA Which statement is true concerning the effect of the application of carburetor heat? A. It enriches the fuel/air mixture. B. It leans the fuel/air mixture. C. It has no effect on the fuel/air mixture. A 2.5.1.9.0.a.1 H51 AIR,RTC,LTA Detonation occurs in a reciprocating aircraft engine when A. there is an explosive increase of fuel caused by too rich a fuel/air mixture. B. the spark plugs receive an electrical jolt caused by a short in the wiring. C. the unburned charge in the cylinders is subjected to instantaneous combustion. C 2.5.2.0.8.a.1 H66 ALL At higher elevation airports the pilot should know that indicated airspeed A. will be unchanged, but groundspeed will be faster. B. will be higher, but groundspeed will be unchanged. C. should be increased to compensate for the thinner air. A 2.5.2.3.4.a.1 H66 AIR,RTC The performance tables of an aircraft for takeoff and climb are based on A. pressure/density altitude. B. cabin altitude. C. true altitude. A 2.5.2.4.0.a.1 H71 RTC Coning is caused by the combined forces of A. drag, weight, and translational lift. B. lift and centrifugal force. C. flapping and centrifugal force. B 2.5.2.4.1.a.1 H71 RTC The forward speed of a rotorcraft is restricted primarily by A. dissymmetry of lift. B. transverse flow effect. C. high-frequency vibrations. A 2.5.2.4.2.a.1 H71 RTC When hovering, a helicopter tends to move in the direction of tail rotor thrust. This statement is A. true; the movement is called transverse tendency. B. true; the movement is called translating tendency. C. false; the movement is opposite the direction of tail rotor thrust, and is called translating tendency. B 2.5.2.4.3.a.1 H71 RTC The purpose of lead-lag (drag) hinges in a three-bladed, fully articulated helicopter rotor system is to compensate for A. Coriolis effect. B. dissymmetry of lift. C. blade flapping tendency. A 2.5.2.4.4.a.1 H71 RTC What happens to the helicopter as it experiences translating tendency? A. It tends to dip slightly to the right as the helicopter approaches approximately 15 knots in takeoff. B. It gains increased rotor efficiency as air over the rotor system reaches approximately 15 knots. C. It moves in the direction of tail rotor thrust. C 2.5.2.4.5.a.1 H71 RTC The unequal lift across the rotor disc that occurs in horizontal flight as a result of the difference in velocity of the air over the advancing half of the disc area and the air passing over the retreating half of the disc area is known as A. coning. B. disc loading. C. dissymmetry of lift. C 2.5.2.4.6.a.1 H71 RTC The lift differential that exists between the advancing blade and the retreating blade is known as A. Coriolis effect. B. translational lift. C. dissymmetry of lift. C 2.5.2.4.7.a.1 H71 RTC Most helicopters, by design tend to drift to the right when hovering in a no-wind condition. This statement is A. false; helicopters have no tendency to drift, but will rotate in that direction. B. true; the mast or cyclic pitch system of most helicopters is rigged forward, this with gyroscopic precession will overcome this tendency. C. true; the mast or cyclic pitch system of most helicopters is rigged to the left to overcome this tendency. C 2.5.2.4.8.a.1 H72 RTC When a rotorcraft transitions from straight-and-level flight into a 30° bank while maintaining a constant altitude, the total lift force must A. increase and the load factor will increase. B. increase and the load factor will decrease. C. remain constant and the load factor will decrease. A 2.5.2.4.9.a.1 H73 RTC Cyclic control pressure is applied during flight that results in a maximum increase in main rotor blade pitch angle at the ``three o'clock'' position. Which way will the rotor disc tilt? A. Aft. B. Left. C. Right. A 2.5.2.5.0.a.1 H73 RTC Cyclic control pressure is applied during flight that results in a maximum decrease in pitch angle of the rotor blades at the ``12 o'clock'' position. Which way will the rotor disc tilt? A. Aft. B. Left. C. Forward. B 2.5.2.5.1.a.1 H73 RTC The primary purpose of the tail rotor system is to A. assist in making coordinated turns. B. maintain heading during forward flight. C. counteract the torque effect of the main rotor. C 2.5.2.5.2.a.1 H73 RTC Can the tail rotor produce thrust to the left? A. No; the right thrust can only be reduced, causing tail movement to the left. B. Yes; primarily so that hovering turns can be accomplished to the right. C. Yes; primarily to counteract the drag of the transmission during autorotation. C 2.5.2.5.3.a.1 H74 RTC The main rotor blades of a fully-articulated rotor system can A. flap and feather collectively. B. flap, drag, and feather independently. C. feather independently, but cannot flap or drag. B 2.5.2.5.4.a.1 H74 RTC A reciprocating engine in a helicopter is more likely to stop due to in-flight carburetor icing than will the same type engine in an airplane. This statement A. has no basis in fact. The same type engine will run equally well in either aircraft. B. is true. The freewheeling unit will not allow windmilling (flywheel) effect to be exerted on a helicopter engine. C. is false. The clutch will immediately release the load from the helicopter engine under engine malfunctioning conditions. B 2.5.2.5.5.a.1 H74 RTC What is the primary purpose of the clutch? A. It allows the engine to be started without driving the main rotor system. B. It provides disengagement of the engine from the rotor system for autorotation. C. It transmits engine power to the main rotor, tail rotor, generator/alternator, and other accessories. A 2.5.2.5.6.a.1 H74 RTC What is the primary purpose of the freewheeling unit? A. It allows the engine to be started without driving the main rotor system. B. It provides speed reduction between the engine, main rotor system, and tail rotor system. C. It provides disengagement of the engine from the rotor system for autorotation purposes. C 2.5.2.5.7.a.1 H74 RTC The main rotor blades of a semirigid rotor system can A. flap and feather as a unit. B. flap, drag, and feather independently. C. feather independently, but cannot flap or drag. A 2.5.2.5.8.a.1 H77 RTC Rotorcraft climb performance is most adversely affected by A. higher than standard temperature and low relative humidity. B. lower than standard temperature and high relative humidity. C. higher than standard temperature and high relative humidity. C 2.5.2.5.9.a.1 H77 RTC The most unfavorable combination of conditions for rotorcraft performance is A. low density altitude, low gross weight, and calm wind. B. high density altitude, high gross weight, and calm wind. C. high density altitude, high gross weight, and strong wind. B 2.5.2.6.0.a.1 H77 RTC How does high density altitude affect rotorcraft performance? A. Engine and rotor efficiency is reduced. B. Engine and rotor efficiency is increased. C. It increases rotor drag, which requires more power for normal flight. A 2.5.2.6.1.a.1 H78 RTC A medium-frequency vibration that suddenly occurs during flight could be indicative of a defective A. main rotor system. B. tail rotor system. C. transmission system. B 2.5.2.6.2.a.1 H78 RTC In most helicopters, medium-frequency vibrations indicate a defective A. engine. B. main rotor system. C. tail rotor system. C 2.5.2.6.3.a.1 H78 RTC Abnormal helicopter vibrations in the low-frequency range are associated with which system or component? A. Tail rotor. B. Main rotor. C. Transmission. B 2.5.2.6.4.a.1 H78 RTC Helicopter low-frequency vibrations are always associated with the A. main rotor. B. tail rotor. C. transmission. A 2.5.2.6.5.a.1 H78 RTC A high-frequency vibration that suddenly occurs during flight could be an indication of a defective A. transmission. B. freewheeling unit. C. main rotor system. A 2.5.2.6.6.a.1 H78 RTC Ground resonance is more likely to occur with helicopters that are equipped with A. rigid rotor systems. B. semi-rigid rotor systems. C. fully articulated rotor systems. C 2.5.2.6.7.a.1 H80 RTC The proper action to initiate a quick stop is to apply A. forward cyclic, while raising the collective and applying right antitorque pedal. B. aft cyclic, while raising the collective and applying left antitorque pedal. C. aft cyclic, while lowering the collective and applying right antitorque pedal. C 2.5.2.7.2.a.1 L34 ALL How can you determine if another aircraft is on a collision course with your aircraft? A. The nose of each aircraft is pointed at the same point in space. B. The other aircraft will always appear to get larger and closer at a rapid rate. C. There will be no apparent relative motion between your aircraft and the other aircraft. C 2.5.2.9.8.a.1 H51 AIR,RTC,LTA The best power mixture is that fuel/air ratio at which A. cylinder head temperatures are the coolest. B. the most power can be obtained for any given throttle setting. C. a given power can be obtained with the highest manifold pressure or throttle setting. B 2.5.2.9.9.a.1 H51 AIR,RTC,LTA Detonation can be caused by A. too lean a mixture. B. low engine temperatures. C. using a higher grade fuel than recommended. A 2.5.3.0.0.a.1 H66 AIR,RTC What effect, if any, would a change in ambient temperature or air density have on gas turbine engine performance? A. As air density decreases, thrust increases. B. As temperature increases, thrust increases. C. As temperature increases, thrust decreases. C 3.5.3.0.1.a.1 I21 ALL Every physical process of weather is accompanied by or is the result of A. a heat exchange. B. the movement of air. C. a pressure differential. A 3.5.3.0.2.a.1 I21 ALL What is the standard temperature at 10,000 feet? A. -5 °C. B. -15 °C. C. +5 °C. A 3.5.3.0.3.a.1 I21 ALL What is the standard temperature at 20,000 feet? A. -15 °C. B. -20 °C. C. -25 °C. C 3.5.3.0.4.a.1 I21 ALL Which conditions are favorable for the formation of a surface based temperature inversion? A. Clear, cool nights with calm or light wind. B. Area of unstable air rapidly transferring heat from the surface. C. Broad areas of cumulus clouds with smooth, level bases at the same altitude. A 3.5.3.0.5.a.1 I22 ALL What are the standard temperature and pressure values for sea level? A. 15 °C and 29.92" Hg. B. 59 °F and 1013.2" Hg. C. 15 °C and 29.92 Mb. A 3.5.3.0.6.a.1 I22 ALL GIVEN: Pressure altitude 12,000 ft True air temperature +50 °F From the conditions given, the approximate density altitude is A. 11,900 feet. B. 14,130 feet. C. 18,150 feet. B 3.5.3.0.7.a.1 I22 ALL GIVEN: Pressure altitude.................5,000 ft True air temperature............+30 °C From the conditions given, the approximate density altitude is A. 7,800 feet. B. 8,100 feet. C. 8,800 feet. A 3.5.3.0.8.a.1 I22 ALL GIVEN: Pressure altitude.................6,000 ft True air temperature............+30 °F From the conditions given, the approximate density altitude is A. 9,000 feet. B. 5,500 feet. C. 5,000 feet. B 3.5.3.0.9.a.1 I22 ALL GIVEN: Pressure altitude.................7,000 ft True air temperature............+15 °C From the conditions given, the approximate density altitude is A. 5,000 feet. B. 8,500 feet. C. 9,500 feet. B 3.5.3.1.0.a.1 I23 ALL What causes wind? A. The Earth's rotation. B. Air mass modification. C. Pressure differences. C 3.5.3.1.1.a.1 I23 ALL In the Northern Hemisphere, the wind is deflected to the A. right by Coriolis force. B. right by surface friction. C. left by Coriolis force. A 3.5.3.1.2.a.1 I23 ALL Why does the wind have a tendency to flow parallel to the isobars above the friction level? A. Coriolis force tends to counterbalance the horizontal pressure gradient. B. Coriolis force acts perpendicular to a line connecting the highs and lows. C. Friction of the air with the Earth deflects the air perpendicular to the pressure gradient. A 3.5.3.1.3.a.1 I23 ALL The wind system associated with a low-pressure area in the Northern Hemisphere is A. an anticyclone and is caused by descending cold air. B. a cyclone and is caused by Coriolis force. C. an anticyclone and is caused by Coriolis force. B 3.5.3.1.4.a.1 I23 ALL With regard to windflow patterns shown on surface analysis charts; when the isobars are A. close together, the pressure gradient force is slight and wind velocities are weaker. B. not close together, the pressure gradient force is greater and wind velocities are stronger. C. close together, the pressure gradient force is greater and wind velocities are stronger. C 3.5.3.1.5.a.1 I23 ALL What prevents air from flowing directly from high-pressure areas to low-pressure areas? A. Coriolis force. B. Surface friction. C. Pressure gradient force. A 3.5.3.1.6.a.1 I23 ALL While flying cross-country, in the Northern Hemisphere, you experience a continuous left crosswind which is associated with a major wind system. This indicates that you A. are flying toward an area of generally unfavorable weather conditions. B. have flown from an area of unfavorable weather conditions. C. cannot determine weather conditions without knowing pressure changes. A 3.5.3.1.7.a.1 I23 ALL Which is true with respect to a high- or low-pressure system? A. A high-pressure area or ridge is an area of rising air. B. A low-pressure area or trough is an area of descending air. C. A high-pressure area or ridge is an area of descending air. C 3.5.3.1.8.a.1 I23 ALL Which is true regarding high- or low-pressure systems? A. A high-pressure area or ridge is an area of rising air. B. A low-pressure area or trough is an area of rising air. C. Both high- and low-pressure areas are characterized by descending air. B 3.5.3.1.9.a.1 I23 ALL When flying into a low-pressure area in the Northern Hemisphere, the wind direction and velocity will be from the A. left and decreasing. B. left and increasing. C. right and decreasing. B 3.5.3.2.0.a.1 I24 ALL Which is true regarding actual air temperature and dewpoint temperature spread? The temperature spread A. decreases as the relative humidity decreases. B. decreases as the relative humidity increases. C. increases as the relative humidity increases. B 3.5.3.2.1.a.1 I24 ALL The general circulation of air associated with a high-pressure area in the Northern Hemisphere is A. outward, downward, and clockwise. B. outward, upward, and clockwise. C. inward, downward, and clockwise. A 3.5.3.2.2.a.1 I24 ALL Virga is best described as A. streamers of precipitation trailing beneath clouds which evaporates before reaching the ground. B. wall cloud torrents trailing beneath cumulonimbus clouds which dissipate before reaching the ground. C. turbulent areas beneath cumulonimbus clouds. A 3.5.3.2.3.a.1 I24 ALL Moisture is added to a parcel of air by A. sublimation and condensation. B. evaporation and condensation. C. evaporation and sublimation. C 3.5.3.2.4.a.1 I24 ALL Ice pellets encountered during flight normally are evidence that A. a warm front has passed. B. a warm front is about to pass. C. there are thunderstorms in the area. B 3.5.3.2.5.a.1 I24 ALL What is indicated if ice pellets are encountered at 8,000 feet? A. Freezing rain at higher altitude. B. You are approaching an area of thunderstorms. C. You will encounter hail if you continue your flight. A 3.5.3.2.6.a.1 I24 ALL Ice pellets encountered during flight are normally evidence that A. a cold front has passed. B. there are thunderstorms in the area. C. freezing rain exists at higher altitudes. C 3.5.3.2.7.a.1 I25 ALL When conditionally unstable air with high-moisture content and very warm surface temperature is forecast, one can expect what type of weather? A. Strong updrafts and stratonimbus clouds. B. Restricted visibility near the surface over a large area. C. Strong updrafts and cumulonimbus clouds. C 3.5.3.2.8.a.1 I25 ALL What is the approximate base of the cumulus clouds if the temperature at 2,000 feet MSL is 70 °F. and the dewpoint is 52 °F? A. 3,000 feet MSL. B. 4,000 feet MSL. C. 6,000 feet MSL. C 3.5.3.2.9.a.1 I25 ALL If clouds form as a result of very stable, moist air being forced to ascend a mountain slope, the clouds will be A. cirrus type with no vertical development or turbulence. B. cumulus type with considerable vertical development and turbulence. C. stratus type with little vertical development and little or no turbulence. C 3.5.3.3.0.a.1 I25 ALL What determines the structure or type of clouds which will form as a result of air being forced to ascend? A. The method by which the air is lifted. B. The stability of the air before lifting occurs. C. The relative humidity of the air after lifting occurs. B 3.5.3.3.1.a.1 I25 ALL Refer to the excerpt from a surface weather report: ABC ...194/89/45/2115/993... At approximately what altitude AGL should bases of convective-type cumuliform clouds be expected? (Use most accurate method.) A. 4,400 feet. B. 10,000 feet. C. 17,600 feet. B 3.5.3.3.2.a.1 I25 ALL What are the characteristics of stable air? A. Good visibility; steady precipitation; stratus clouds. B. Poor visibility; steady precipitation; stratus clouds. C. Poor visibility; intermittent precipitation; cumulus clouds. B 3.5.3.3.3.a.1 I25 ALL Which would decrease the stability of an air mass? A. Warming from below. B. Cooling from below. C. Decrease in water vapor. A 3.5.3.3.4.a.1 I25 ALL From which measurement of the atmosphere can stability be determined? A. Atmospheric pressure. B. The ambient lapse rate. C. The dry adiabatic lapse rate. B 3.5.3.3.5.a.1 I25 ALL What type weather can one expect from moist, unstable air, and very warm surface temperatures? A. Fog and low stratus clouds. B. Continuous heavy precipitation. C. Strong updrafts and cumulonimbus clouds. C 3.5.3.3.6.a.1 I25 ALL Which would increase the stability of an air mass? A. Warming from below. B. Cooling from below. C. Decrease in water vapor. B 3.5.3.3.7.a.1 I26 ALL The conditions necessary for the formation of stratiform clouds are a lifting action and A. unstable, dry air. B. stable, moist air. C. unstable, moist air. B 3.5.3.3.8.a.1 I26 ALL Which cloud types would indicate convective turbulence? A. Cirrus clouds. B. Nimbostratus clouds. C. Towering cumulus clouds. C 3.5.3.3.9.a.1 I26 ALL The presence of standing lenticular altocumulus clouds is a good indication of A. lenticular ice formation in calm air. B. very strong turbulence. C. heavy icing conditions. B 3.5.3.4.0.a.1 I27 ALL The formation of either predominantly stratiform or predominantly cumuliform clouds is dependent upon the A. source of lift. B. stability of the air being lifted. C. temperature of the air being lifted. B 3.5.3.4.1.a.1 I27 ALL Which combination of weather-producing variables would likely result in cumuliform-type clouds, good visibility, and showery rain? A. Stable, moist air and orographic lifting. B. Unstable, moist air and orographic lifting. C. Unstable, moist air and no lifting mechanism. B 3.5.3.4.2.a.1 I27 ALL What is a characteristic of stable air? A. Stratiform clouds. B. Fair weather cumulus clouds. C. Temperature decreases rapidly with altitude. A 3.5.3.4.3.a.1 I27 ALL A moist, unstable air mass is characterized by A. poor visibility and smooth air. B. cumuliform clouds and showery precipitation. C. stratiform clouds and continuous precipitation. B 3.5.3.4.4.a.1 I27 ALL When an air mass is stable, which of these conditions are most likely to exist? A. Numerous towering cumulus and cumulonimbus clouds. B. Moderate to severe turbulence at the lower levels. C. Smoke, dust, haze, etc., concentrated at the lower levels with resulting poor visibility. C 3.5.3.4.5.a.1 I27 ALL Which is a characteristic of stable air? A. Cumuliform clouds. B. Excellent visibility. C. Restricted visibility. C 3.5.3.4.6.a.1 I27 ALL Which is a characteristic typical of a stable air mass? A. Cumuliform clouds. B. Showery precipitation. C. Continuous precipitation. C 3.5.3.4.7.a.1 I27 ALL Which is true regarding a cold front occlusion? The air ahead of the warm front A. is colder than the air behind the overtaking cold front. B. is warmer than the air behind the overtaking cold front. C. has the same temperature as the air behind the overtaking cold front. B 3.5.3.4.8.a.1 I27 ALL Which are characteristics of a cold air mass moving over a warm surface? A. Cumuliform clouds, turbulence, and poor visibility. B. Cumuliform clouds, turbulence, and good visibility. C. Stratiform clouds, smooth air, and poor visibility. B 3.5.3.4.9.a.1 I27 ALL The conditions necessary for the formation of cumulonimbus clouds are a lifting action and A. unstable, dry air. B. stable, moist air. C. unstable, moist air. C 3.5.3.5.0.a.1 I27 ALL Fog produced by frontal activity is a result of saturation due to A. nocturnal cooling. B. adiabatic cooling. C. evaporation of precipitation. C 3.5.3.5.1.a.1 I28 ALL What is an important characteristic of wind shear? A. It is present at only lower levels and exists in a horizontal direction. B. It is present at any level and exists in only a vertical direction. C. It can be present at any level and can exist in both a horizontal and vertical direction. C 3.5.3.5.2.a.1 I28 ALL Hazardous wind shear is commonly encountered A. near warm or stationary frontal activity. B. when the wind velocity is stronger than 35 knots. C. in areas of temperature inversion and near thunderstorms. C 3.5.3.5.3.a.1 I28 ALL Low-level wind shear may occur when A. surface winds are light and variable. B. there is a low-level temperature inversion with strong winds above the inversion. C. surface winds are above 15 knots and there is no change in wind direction and windspeed with height. B 3.5.3.5.4.a.1 I28 ALL If a temperature inversion is encountered immediately after takeoff or during an approach to a landing, a potential hazard exists due to A. wind shear. B. strong surface winds. C. strong convective currents. A 3.5.3.5.5.a.1 I28 ALL GIVEN: Winds at 3,000 feet AGL....................30 kts Surface winds......................................Calm While approaching for landing under clear skies a few hours after sunrise, one should A. allow a margin of approach airspeed above normal to avoid stalling. B. keep the approach airspeed at or slightly below normal to compensate for floating. C. not alter our approach airspeed, these conditions are nearly ideal. A 3.5.3.5.6.a.1 I28 ALL Convective currents are most active on warm summer afternoons when winds are A. light. B. moderate. C. strong. A 3.5.3.5.7.a.1 I28 ALL When flying low over hilly terrain, ridges, or mountain ranges, the greatest potential danger from turbulent air currents will usually be encountered on the A. leeward side when flying with a tailwind. B. leeward side when flying into the wind. C. windward side when flying into the wind. B 3.5.3.5.8.a.1 I28 AIR,RTC During an approach, the most important and most easily recognized means of being alerted to possible wind shear is monitoring the A. amount of trim required to relieve control pressures. B. heading changes necessary to remain on the runway centerline. C. power and vertical velocity required to remain on the proper glidepath. C 3.5.3.5.9.a.1 I28 ALL During departure, under conditions of suspected low-level wind shear, a sudden decrease in headwind will cause A. a loss in airspeed equal to the decrease in wind velocity. B. a gain in airspeed equal to the decrease in wind velocity. C. no change in airspeed, but groundspeed will decrease. A 3.5.3.6.0.a.1 I29 ALL Which situation would most likely result in freezing precipitation? Rain falling from air which has a temperature of A. 32 °F or less into air having a temperature of more than 32 °F. B. 0 °C or less into air having a temperature of 0 °C or more. C. more than 32 °F into air having a temperature of 32 °F or less. C 3.5.3.6.1.a.1 I30 ALL Which statement is true concerning the hazards of hail? A. Hail damage in horizontal flight is minimal due to the vertical movement of hail in the clouds. B. Rain at the surface is a reliable indication of no hail aloft. C. Hailstones may be encountered in clear air several miles from a thunderstorm. C 3.5.3.6.2.a.1 I30 ALL Hail is most likely to be associated with A. cumulus clouds. B. cumulonimbus clouds. C. stratocumulus clouds. B 3.5.3.6.3.a.1 I30 ALL The most severe weather conditions, such as destructive winds, heavy hail, and tornadoes, are generally associated with A. slow-moving warm fronts which slope above the tropopause. B. squall lines. C. fast-moving occluded fronts. B 3.5.3.6.4.a.1 I30 ALL Of the following, which is accurate regarding turbulence associated with thunderstorms? A. Outside the cloud, shear turbulence can be encountered 50 miles laterally from a severe storm. B. Shear turbulence is encountered only inside cumulonimbus clouds or within a 5-mile radius of them. C. Outside the cloud, shear turbulence can be encountered 20 miles laterally from a severe storm. C 3.5.3.6.5.a.1 I30 ALL If airborne radar is indicating an extremely intense thunderstorm echo, this thunderstorm should be avoided by a distance of at least A. 20 miles. B. 10 miles. C. 5 miles. A 3.5.3.6.6.a.1 I30 ALL Which statement is true regarding squall lines? A. They are always associated with cold fronts. B. They are slow in forming, but rapid in movement. C. They are nonfrontal and often contain severe, steady-state thunderstorms. C 3.5.3.6.7.a.1 I30 ALL Which statement is true concerning squall lines? A. They form slowly, but move rapidly. B. They are associated with frontal systems only. C. They offer the most intense weather hazards to aircraft. C 3.5.3.6.8.a.1 I30 ALL Select the true statement pertaining to the life cycle of a thunderstorm. A. Updrafts continue to develop throughout the dissipating stage of a thunderstorm. B. The beginning of rain at the Earth's surface indicates the mature stage of the thunderstorm. C. The beginning of rain at the Earth's surface indicates the dissipating stage of the thunderstorm. B 3.5.3.6.9.a.1 I30 ALL What visible signs indicate extreme turbulence in thunderstorms? A. Base of the clouds near the surface, heavy rain, and hail. B. Low ceiling and visibility, hail, and precipitation static. C. Cumulonimbus clouds, very frequent lightning, and roll clouds. C 3.5.3.7.0.a.1 I30 ALL Which weather phenomenon signals the beginning of the mature stage of a thunderstorm? A. The start of rain. B. The appearance of an anvil top. C. Growth rate of cloud is maximum. A 3.5.3.7.1.a.1 I30 ALL What feature is normally associated with the cumulus stage of a thunderstorm? A. Roll cloud. B. Continuous updraft. C. Beginning of rain at the surface. B 3.5.3.7.2.a.1 I30 ALL During the life cycle of a thunderstorm, which stage is characterized predominately by downdrafts? A. Mature. B. Developing. C. Dissipating. C 3.5.3.7.3.a.1 I30 ALL What minimum distance should exist between intense radar echoes before any attempt is made to fly between these thunderstorms? A. 20 miles. B. 30 miles. C. 40 miles. C 3.5.3.7.4.a.1 I31 ALL Which in-flight hazard is most commonly associated with warm fronts? A. Advection fog. B. Radiation fog. C. Precipitation-induced fog. C 3.5.3.7.5.a.1 I31 ALL Which is true regarding the use of airborne weather-avoidance radar for the recognition of certain weather conditions? A. The radarscope provides no assurance of avoiding instrument weather conditions. B. The avoidance of hail is assured when flying between and just clear of the most intense echoes. C. The clear area between intense echoes indicates that visual sighting of storms can be maintained when flying between the echoes. A 3.5.3.7.6.a.1 I31 ALL A situation most conducive to the formation of advection fog is A. a light breeze moving colder air over a water surface. B. an air mass moving inland from the coastline during the winter. C. a warm, moist air mass settling over a cool surface under no-wind conditions. B 3.5.3.7.7.a.1 I31 ALL Advection fog has drifted over a coastal airport during the day. What may tend to dissipate or lift this fog into low stratus clouds? A. Nighttime cooling. B. Surface radiation. C. Wind 15 knots or stronger. C 3.5.3.7.8.a.1 I31 ALL What lifts advection fog into low stratus clouds? A. Nighttime cooling. B. Dryness of the underlying land mass. C. Surface winds of approximately 15 knots or stronger. C 3.5.3.7.9.a.1 I31 ALL In what ways do advection fog, radiation fog, and steam fog differ in their formation or location? A. Radiation fog is restricted to land areas; advection fog is most common along coastal areas; steam fog forms over a water surface. B. Advection fog deepens as windspeed increases up to 20 knots; steam fog requires calm or very light wind; radiation fog forms when the ground or water cools the air by radiation. C. Steam fog forms from moist air moving over a colder surface; advection fog requires cold air over a warmer surface; radiation fog is produced by radiational cooling of the ground. A 3.5.3.8.0.a.1 I31 ALL With respect to advection fog, which statement is true? A. It is slow to develop, and dissipates quite rapidly. B. It forms almost exclusively at night or near daybreak. C. It can appear suddenly during day or night, and it is more persistent than radiation fog. C 3.5.3.8.1.a.1 I32 ALL Which feature is associated with the tropopause? A. Constant height above the Earth. B. Abrupt change in temperature lapse rate. C. Absolute upper limit of cloud formation. B 3.5.3.8.2.a.1 I32 ALL A common location of clear air turbulence is A. in an upper trough on the polar side of a jet stream. B. near a ridge aloft on the equatorial side of a high-pressure flow. C. south of an east/west oriented high-pressure ridge in its dissipating stage. A 3.5.3.8.3.a.1 I32 ALL The jet stream and associated clear air turbulence can sometimes be visually identified in flight by A. dust or haze at flight level. B. long streaks of cirrus clouds. C. a constant outside air temperature. B 3.5.3.8.4.a.1 I32 ALL During the winter months in the middle latitudes, the jet stream shifts toward the A. north and speed decreases. B. south and speed increases. C. north and speed increases. B 3.5.3.8.5.a.1 I32 ALL The strength and location of the jet stream is normally A. weaker and farther north in the summer. B. stronger and farther north in the winter. C. stronger and farther north in the summer. A 3.5.3.9.2.a.1 I35 ALL Convective circulation patterns associated with sea breezes are caused by A. water absorbing and radiating heat faster than the land. B. land absorbing and radiating heat faster than the water. C. cool and less dense air moving inland from over the water, causing it to rise. B 3.5.3.9.3.a.1 I35 ALL The conditions most favorable to wave formation over mountainous areas are a layer of A. stable air at mountaintop altitude and a wind of at least 20 knots blowing across the ridge. B. unstable air at mountaintop altitude and a wind of at least 20 knots blowing across the ridge. C. moist, unstable air at mountaintop altitude and a wind of less than 5 knots blowing across the ridge. A 3.5.3.9.8.a.1 I40 ALL During preflight preparation, weather report forecasts which are not routinely available at the local service outlet (FSS or WSFO) can best be obtained by means of the A. request/reply service. B. air route traffic control center. C. pilot's automatic telephone answering service. A 3.5.3.9.9.a.1 I40 ALL The most current en route and destination weather information for an instrument flight should be obtained from A. the FSS or WSO. B. the ATIS broadcast. C. NOTAM's (Class II). A 3.5.4.0.0.a.1 I40 ALL FSS's in the conterminous 48 U.S. having voice capability on VOR's or radiobeacons (NDB's) broadcast A. AIRMET's and SIGMET's at 15 minutes past the hour and each 15 minutes thereafter as long as they are in effect. B. AIRMET's and Nonconvective SIGMET's at 15 minutes and 45 minutes past the hour for the first hour after issuance. C. hourly weather reports at 15 and 45 minutes past each hour for those reporting stations within approximately 150 NM of the broadcast stations. B 3.5.4.0.1.a.1 I40 ALL Transcribed Weather Broadcasts (TWEB's) may be monitored by tuning the appropriate radio receiver to certain A. NDB, but not VOR frequencies. B. VOR and NDB frequencies. C. VOR, but not NDB frequencies. B 3.5.4.0.2.a.1 I41 ALL The remarks section of the hourly aviation weather report contains the following coded information: RADAT 87045 What is the meaning of this information? A. Radar echoes with tops at 45,000 feet were observed on the 087 radial of the VORTAC. B. A pilot reported thunderstorms 87 DME miles distance on the 045 radial of the VORTAC. C. Relative humidity was 87 percent and the freezing level (0 °C) was at 4,500 feet MSL. C 3.5.4.0.3.a.1 I41 ALL What is meant by the entry in the remarks section of this Surface Aviation Weather Report for BOI? BOI SP 1854 -X M7 OVC 1 1/2R+F 990/63/61/ 3205/980/RF2 RB12 A. Rain and fog obscuring two-tenths of the sky; rain began at 1912. B. Rain and fog obscuring two-tenths of the sky; rain began at 1812. C. Runway fog, visibility 2 miles; base of the rain clouds 1,200 feet. B 3.5.4.0.4.a.1 I41 ALL The station originating the following weather report has a field elevation of 3,500 feet MSL. If the sky cover is one continuous layer, what is its thickness? M5 OVC 1/2HK 173/73/72/0000/002/OVC 75 A. 2,500 feet. B. 3,500 feet. C. 4,000 feet. B 3.5.4.0.5.a.1 I41 ALL What wind conditions would you anticipate when squalls are reported at your destination? A. Rapid variations in windspeed of 15 knots or more between peaks and lulls. B. Peak gusts of at least 35 knots combined with a change in wind direction of 30° or more. C. Sudden increases in windspeed of at least 15 knots to a sustained speed of 20 knots or more for at least 1 minute. C 3.5.4.0.6.a.1 I41 ALL What significant cloud coverage is reported by a pilot in this SA? MOB...M9 OVC 2LF 131/44/43/3212/991/UA/OV 15NW MOB 1355/SK OVC 025/045 OVC 090 A. Three separate overcast layers exist with bases at 2,500, 7,500, and 13,500 feet. B. The top of lower overcast is 2,500 feet; base and top of second overcast layer is 4,500 and 9,000 feet, respectively. C. The base of second overcast layer is 2,500 feet; top of second overcast layer is 7,500 feet; base of third layer is 13,500 feet. B 3.5.4.0.7.a.1 I42 ALL To best determine observed weather conditions between weather reporting stations, the pilot should refer to A. pilot reports. B. Area Forecasts. C. prognostic charts. A 3.5.4.0.8.a.1 I42 ALL Which is true concerning this radar weather report for OKC? OKC 1934 LN 8TRW+/+ 86/40 164/60 199/115 15W 2425 MT 570 AT 159/65 2 INCH HAIL RPRTD THIS ECHO A. There are three cells with tops at 11,500, 40,000, and 60,000 feet. B. The line of cells is moving 080° with winds reported up to 40 knots. C. The maximum top of the cells is 57,000 feet located 65 NM south-southeast of the station. C 3.5.4.0.9.a.1 I43 ALL What is the meaning of the term MVFR, as used in the categorical outlook portion of Terminal and Area Forecasts? A. A ceiling less than 1,000 feet, and/or visibility less than 3 miles. B. A ceiling of 1,000 to 3,000 feet, and/or visibility of 3 to 5 miles. C. A ceiling of 3,000 to 5,000 feet, and visibility of 5 to 7 miles. B 3.5.4.1.0.a.1 I43 ALL The contraction WND in the 6-hour categorical outlook in the Terminal Forecast means that the wind during that period is forecast to be A. 15 to 20 knots. B. less than 25 knots. C. 25 knots or stronger. C 3.5.4.1.1.a.1 I43 ALL Which statement pertaining to a Terminal Forecast is true? The term A. WND in the categorical outlook implies surface winds are forecast to be 10 knots or greater. B. CHC TRW VCNTY in the remarks section pertains to an area within a 5-mile radius of the airport. C. VFR CIGS ABV 100 in the categorical outlook implies ceilings above 10,000 feet and visibility more than 5 miles. C 3.5.4.1.2.a.1 I43 ALL The absence of a visibility entry in a Terminal Forecast specifically implies that the surface visibility is expected to be more than A. 3 miles. B. 6 miles. C. 10 miles. B 3.5.4.1.3.a.1 I43 ALL Terminal Forecasts are issued how many times a day and cover what period of time? A. Three times daily and are valid for 24 hours including a 6-hour categorical outlook. B. Four times daily and are valid for 18 hours including a 4-hour categorical outlook. C. Six times daily and are valid for 12 hours with an additional 6-hour categorical outlook. A 3.5.4.1.4.a.1 I43 ALL Which information is contained in the HAZARDS section of the Area Forecast? A. A summary of general weather conditions for the entire region covered in the Area Forecast. B. A brief list of weather phenomena that meet AIRMET and/or SIGMET criteria and the location of each. C. A brief summary of significant weather and clouds that do not meet AIRMET, but meet SIGMET criteria. B 3.5.4.1.5.a.1 I43 ALL The section of the Area Forecast entitled SGFNT CLOUD AND WX contains a summary of A. forecast sky cover, cloud tops, visibility, and obstructions to vision along specific routes. B. only those weather systems producing liquid or frozen precipitation, fog, thunderstorms, or IFR ceilings. C. sky condition, cloud heights, visibility, weather and/or obstructions to visibility, and surface winds of 30 knots or more. C 3.5.4.1.6.a.1 I43 ALL In the HAZARDS AND FLIGHT PRECAUTIONS section of an Area Forecast, what is indicated by the forecast term - - FLT PRCTNS...IFR...TX AR LA MS TN AL AND CSTL WTRS? A. IFR conditions which meet in-flight advisory criteria are forecast for the states listed. B. Each state and geographic area listed is reporting ceilings and visibilities below VFR minimums. C. IFR conditions, turbulence, and icing are all forecast within the valid period for the listed states. A 3.5.4.1.7.a.1 I43 ALL In the Area Forecast (FA), what method is used to describe the location of each icing phenomenon? A. VOR points outline the affected area(s) within the designated FA boundary, but not beyond the FA boundary. B. State names and portions of states, such as northwest and south central, are used to outline each affected area. C. VOR points are used to outline the area of icing, including VOR points outside the designated FA boundary, if necessary. C 3.5.4.1.8.a.1 I43 ALL What single reference contains information regarding expected frontal movement, turbulence, and icing conditions for a specific area? A. Area Forecast. B. Surface Analysis Chart. C. Weather Depiction Chart. A 3.5.4.1.9.a.1 I43 ALL The National Aviation Weather Advisory Unit prepares FA's for the contiguous U.S. A. twice each day. B. three times each day. C. every 6 hours unless significant changes in weather require it more often. B 3.5.4.2.0.a.1 I43 ALL Which forecast provides specific information concerning expected sky cover, cloud tops, visibility, weather, and obstructions to vision in a route format? A. Area Forecast. B. Terminal Forecast. C. Transcribed Weather Broadcast. C 3.5.4.2.1.a.1 I43 ALL To obtain a continuous transcribed weather briefing including winds aloft and route forecasts for a cross-country flight, a pilot could monitor A. a TWEB on a low-frequency radio receiver. B. the regularly scheduled weather broadcast on a VOR frequency. C. a high-frequency radio receiver tuned to En Route Flight Advisory Service. A 3.5.4.2.2.a.1 I43 ALL SIGMET's are issued as a warning of weather conditions which are hazardous A. to all aircraft. B. particularly to heavy aircraft. C. particularly to light airplanes. A 3.5.4.2.3.a.1 I43 ALL Which correctly describes the purpose of convective SIGMET's (WST)? A. They consist of an hourly observation of tornadoes, significant thunderstorm activity, and large hailstone activity. B. They contain both an observation and a forecast of all thunderstorm and hailstone activity. The forecast is valid for 1 hour only. C. They consist of either an observation and a forecast or just a forecast for tornadoes, significant thunderstorm activity, or hail greater than or equal to 3/4 inch in diameter. C 3.5.4.2.4.a.1 I43 ALL What values are used for Winds Aloft Forecasts? A. True direction and MPH. B. True direction and knots. C. Magnetic direction and knots. B 3.5.4.2.5.a.1 I43 ALL On a Surface Analysis Chart, the solid lines that depict sea level pressure patterns are called A. isobars. B. isogons. C. millibars. A 3.5.4.2.6.a.1 I44 ALL Dashed lines on a Surface Analysis Chart, if depicted, indicate that the pressure gradient is A. weak. B. strong. C. unstable. A 3.5.4.2.7.a.1 I44 ALL Which chart provides a ready means of locating observed frontal positions and pressure centers? A. Surface Analysis Chart. B. Constant Pressure Analysis Chart. C. Weather Depiction Chart. A 3.5.4.2.8.a.1 I44 ALL On a Surface Analysis Chart, close spacing of the isobars indicates A. weak pressure gradient. B. strong pressure gradient. C. strong temperature gradient. B 3.5.4.2.9.a.1 I44 ALL The Surface Analysis Chart depicts A. frontal locations and expected movement, pressure centers, cloud coverage, and obstructions to vision at the time of chart transmission. B. actual frontal positions, pressure patterns, temperature, dewpoint, wind, weather, and obstructions to vision at the valid time of the chart. C. actual pressure distribution, frontal systems, cloud heights and coverage, temperature, dewpoint, and wind at the time shown on the chart. B 3.5.4.3.0.a.1 I45 ALL Which provides a graphic display of both VFR and IFR weather? A. Surface Weather Map. B. Radar Summary Chart. C. Weather Depiction Chart. C 3.5.4.3.1.a.1 I45 ALL When total sky cover is few or scattered, the height shown on the Weather Depiction Chart is the A. top of the lowest layer. B. base of the lowest layer. C. base of the highest layer. B 3.5.4.3.2.a.1 I46 ALL What information is provided by the Radar Summary Chart that is not shown on other weather charts? A. Lines and cells of hazardous thunderstorms. B. Ceilings and precipitation between reporting stations. C. Areas of cloud cover and icing levels within the clouds. A 3.5.4.3.3.a.1 I47 ALL Which weather chart depicts conditions forecast to exist at a specific time in the future? A. Freezing Level Chart. B. Weather Depiction Chart. C. 12-Hour Significant Weather Prognostication Chart. C 3.5.4.3.4.a.1 I47 ALL What weather phenomenon is implied within an area enclosed by small scalloped lines on a U.S. High-Level Significant Weather Prognostic Chart? A. Cirriform clouds, light to moderate turbulence, and icing. B. Cumulonimbus clouds, icing, and moderate or greater turbulence. C. Cumuliform or standing lenticular clouds, moderate to severe turbulence, and icing. B 3.5.4.3.5.a.1 I47 ALL The U.S. High-Level Significant Weather Prognostic Chart forecasts significant weather for what airspace? A. 18,000 feet to 45,000 feet. B. 24,000 feet to 45,000 feet. C. 24,000 feet to 63,000 feet. C 3.5.4.3.6.a.1 I47 ALL What is the upper limit of the Low Level Significant Weather Prognostic Chart? A. 30,000 feet. B. 24,000 feet. C. 18,000 feet. B 3.5.4.3.8.a.1 I49 ALL A freezing level panel of the composite moisture stability chart is an analysis of A. forecast freezing level data from surface observations. B. forecast freezing level data from upper air observations. C. observed freezing level data from upper air observations. C 3.5.4.3.9.a.1 I49 ALL The difference found by subtracting the temperature of a parcel of air theoretically lifted from the surface to 500 millibars and the existing temperature at 500 millibars is called the A. lifted index. B. negative index. C. positive index. A 3.5.4.4.0.a.1 I51 ALL Hatching on a Constant Pressure Analysis Chart indicates A. hurricane eye. B. windspeed 70 knots to 110 knots. C. windspeed 110 knots to 150 knots. B 3.5.4.4.1.a.1 I51 ALL What flight planning information can a pilot derive from Constant Pressure Analysis Charts? A. Winds and temperatures aloft. B. Clear air turbulence and icing conditions. C. Frontal systems and obstructions to vision aloft. A 3.5.4.4.2.a.1 I51 ALL From which of the following can the observed temperature, wind, and temperature/ dewpoint spread be determined at a specified altitude? A. Stability Charts. B. Winds Aloft Forecasts. C. Constant Pressure Analysis Charts. C 3.5.4.4.3.a.1 I52 ALL The minimum vertical wind shear value critical for probable moderate or greater turbulence is A. 4 knots per 1,000 feet. B. 6 knots per 1,000 feet. C. 8 knots per 1,000 feet. B 3.5.4.4.4.a.1 I53 ALL A pilot reporting turbulence that momentarily causes slight, erratic changes in altitude and/or attitude should report it as A. light chop. B. light turbulence. C. moderate turbulence. B 3.5.4.4.5.a.1 I53 ALL When turbulence causes changes in altitude and/or attitude, but aircraft control remains positive, that should be reported as A. light. B. severe. C. moderate. C 3.5.4.4.6.a.1 I53 ALL Turbulence that is encountered above 15,000 feet AGL not associated with cumuliform cloudiness, including thunderstorms, should be reported as A. severe turbulence. B. clear air turbulence. C. convective turbulence. B 3.5.4.4.7.a.1 K02 ALL Which type of jetstream can be expected to cause the greater turbulence? A. A straight jetstream associated with a low-pressure trough. B. A curving jetstream associated with a deep low-pressure trough. C. A jetstream occurring during the summer at the lower latitudes. B 3.5.4.4.8.a.1 K02 ALL A strong wind shear can be expected A. in the jetstream front above a core having a speed of 60 to 90 knots. B. if the 5 °C isotherms are spaced between 7° to 10° of latitude. C. on the low-pressure side of a jetstream core where the speed at the core is stronger than 110 knots. C 3.5.4.4.9.a.1 J33 ALL Low-level wind shear is best described as a A. violently rotating column of air extending from a cumulonimbus cloud. B. change in wind direction and/or speed within a very short distance in the atmosphere. C. downward motion of the air associated with continuous winds blowing with an easterly component due to the rotation of the Earth. B 3.5.4.5.0.a.1 N33 ALL One of the most dangerous features of mountain waves is the turbulent areas in and A. below rotor clouds. B. above rotor clouds. C. below lenticular clouds. A 4.5.4.7.5.a.1 H06 AIR,RTC GIVEN: True course 105° True heading 085° True airspeed 95 kts Groundspeed 87 kts Determine the wind direction and speed. A. 020° and 32 knots. B. 030° and 38 knots. C. 200° and 32 knots. A 4.5.4.7.6.a.1 H06 AIR,RTC GIVEN: True course 345° True heading 355° True airspeed 85 kts Groundspeed 95 kts Determine the wind direction and speed. A. 095° and 19 knots. B. 113° and 19 knots. C. 238° and 18 knots. B 4.5.4.7.7.a.1 H06 AIR,RTC You have flown 52 miles, are 6 miles off course, and have 118 miles yet to fly. To converge on your destination, the total correction angle would be A. 3°. B. 6°. C. 10°. C 4.5.4.7.8.a.1 H06 ALL GIVEN: Distance off course...............................9 mi Distance flown....................................95 mi Distance to fly...................................125 mi To converge at the destination, the total correction angle would be A. 4°. B. 6°. C. 10°. C 4.5.4.7.9.a.1 H07 ALL True course measurements on a Sectional Aeronautical Chart should be made at a meridian near the midpoint of the course because the A. values of isogonic lines change from point to point. B. angles formed by isogonic lines and lines of latitude vary from point to point. C. angles formed by lines of longitude and the course line vary from point to point. C 4.5.4.8.1.a.1 H07 AIR,RTC GIVEN: Wind.....................................175° at 20 kts Distance .........................................135 NM True course..........................................075° True airspeed.....................................80 kts Fuel consumption..........................105 lb/hr Determine the time en route and fuel consumption. A. 1 hour 28 minutes and 73.2 pounds. B. 1 hour 38 minutes and 158 pounds. C. 1 hour 40 minutes and 175 pounds. C 4.5.4.9.0.a.1 H07 ALL Which is true about homing when using ADF during crosswind conditions? Homing A. to a radio station results in a curved path that leads to the station. B. is a practical navigation method for flying both to and from a radio station. C. to a radio station requires that the ADF have an automatically or manually rotatable azimuth. A 4.5.4.9.1.a.1 H07 ALL Which is true regarding tracking on a desired bearing when using ADF during crosswind conditions? A. To track outbound, heading corrections should be made away from the ADF pointer. B. When on the desired track outbound with the proper drift correction established, the ADF pointer will be deflected to the windward side of the tail position. C. When on the desired track inbound with the proper drift correction established, the ADF pointer will be deflected to the windward side of the nose position. B 4.5.4.9.2.a.1 H07 ALL An aircraft is maintaining a magnetic heading of 265° and the ADF shows a relative bearing of 065°. This indicates that the aircraft is crossing the A. 065° magnetic bearing FROM the radio beacon. B. 150° magnetic bearing FROM the radio beacon. C. 330° magnetic bearing FROM the radio beacon. B 4.5.4.9.3.a.1 H07 ALL The magnetic heading is 315° and the ADF shows a relative bearing of 140°. The magnetic bearing FROM the radiobeacon would be A. 095°. B. 175°. C. 275°. C 4.5.4.9.4.a.1 H07 ALL The magnetic heading is 350° and the relative bearing to a radiobeacon is 240°. What would be the magnetic bearing TO that radiobeacon? A. 050°. B. 230°. C. 295°. B 4.5.4.9.5.a.1 H07 ALL The ADF is tuned to a radiobeacon. If the magnetic heading is 040° and the relative bearing is 290°, the magnetic bearing TO that radiobeacon would be A. 150°. B. 285°. C. 330°. C 4.5.4.9.6.a.1 H07 ALL If the relative bearing to a nondirectional radiobeacon is 045° and the magnetic heading is 355°, the magnetic bearing TO that radiobeacon would be A. 040°. B. 065°. C. 220°. A 4.5.4.9.7.a.1 H07 ALL (Refer to figure 16.) If the aircraft continues its present heading as shown in instrument group 3, what will be the relative bearing when the aircraft reaches the magnetic bearing of 030° FROM the NDB? A. 030°. B. 060°. C. 240°. C 4.5.4.9.8.a.1 H07 ALL (Refer to figure 16.) At the position indicated by instrument group 1, what would be the relative bearing if the aircraft were turned to a magnetic heading of 090°? A. 150°. B. 190°. C. 250°. C 4.5.4.9.9.a.1 H07 ALL (Refer to figure 16.) At the position indicated by instrument group 1, to intercept the 330° magnetic bearing to the NDB at a 30° angle, the aircraft should be turned A. left to a heading of 270°. B. right to a heading of 330°. C. right to a heading of 360°. C 4.5.5.0.0.a.1 H07 ALL Which situation would result in reverse sensing of a VOR receiver? A. Flying a heading that is reciprocal to the bearing selected on the OBS. B. Setting the OBS to a bearing that is 90° from the bearing on which the aircraft is located. C. Failing to change the OBS from the selected inbound course to the outbound course after passing the station. A 4.5.5.0.1.a.1 H07 ALL To track outbound on the 180 radial of a VOR station, the recommended procedure is to set the OBS to A. 360° and make heading corrections toward the CDI needle. B. 180° and make heading corrections away from the CDI needle. C. 180° and make heading corrections toward the CDI needle. C 4.5.5.0.2.a.1 H07 ALL To track inbound on the 215 radial of a VOR station, the recommended procedure is to set the OBS to A. 215° and make heading corrections toward the CDI needle. B. 215° and make heading corrections away from the CDI needle. C. 035° and make heading corrections toward the CDI needle. C 4.5.5.0.3.a.1 H61 ALL When diverting to an alternate airport because of an emergency, pilots should A. rely upon radio as the primary method of navigation. B. climb to a higher altitude because it will be easier to identify checkpoints. C. apply rule-of-thumb computations, estimates, and other appropriate shortcuts to divert to the new course as soon as possible. C 4.5.5.0.4.a.1 H61 ALL To use VHF/DF facilities for assistance in locating your position, you must have an operative VHF A. transmitter and receiver. B. transmitter and receiver, and an operative ADF receiver. C. transmitter and receiver, and an operative VOR receiver. A 4.5.5.0.5.a.1 H66 ALL Which maximum range factor decreases as weight decreases? A. Altitude. B. Airspeed. C. Angle of attack. B 4.5.5.0.6.a.1 I04 ALL (Refer to figure 17.) Which illustration indicates that the airplane will intercept the 360 radial at a 60° angle inbound, if the present heading is maintained? A. 3. B. 4. C. 5. A 4.5.5.0.7.a.1 I04 AIR,RTC,LTA (Refer to figure 17.) Which statement is true regarding illustration 2, if the present heading is maintained? The airplane will A. cross the 180 radial at a 45° angle outbound. B. intercept the 225 radial at a 45° angle. C. intercept the 360 radial at a 45° angle inbound. X/C 4.5.5.0.8.a.1 I04 AIR,RTC,LTA (Refer to figure 17.) Which illustration indicates that the airplane will intercept the 060 radial at a 75° angle outbound, if the present heading is maintained? A. 4. B. 5. C. 6. B 4.5.5.0.9.a.1 I04 AIR,RTC,LTA (Refer to figure 17.) Which illustration indicates that the airplane should be turned 150° left to intercept the 360 radial at a 60° angle inbound? A. 1. B. 2. C. 3. A 4.5.5.1.0.a.1 I04 AIR,RTC,LTA (Refer to figure 17.) Which is true regarding illustration 4, if the present heading is maintained? The airplane will A. cross the 060 radial at a 15° angle. B. intercept the 240 radial at a 30° angle. C. cross the 180 radial at a 75° angle. C 4.5.5.1.1.a.1 I08 AIR,RTC,LTA (Refer to figure 18.) To intercept a magnetic bearing of 240° FROM at a 030° angle (while outbound), the airplane should be turned A. left 065°. B. left 125°. C. right 270°. B 4.5.5.1.2.a.1 I08 AIR,RTC,LTA (Refer to figure 18.) If the airplane continues to fly on the heading as shown, what magnetic bearing FROM the station would be intercepted at a 35°angle outbound? A. 035°. B. 070°. C. 215°. B 4.5.5.1.3.a.1 I08 AIR,RTC,LTA (Refer to figure 19.) If the airplane continues to fly on the magnetic heading as illustrated, what magnetic bearing FROM the station would be intercepted at a 35°angle? A. 090°. B. 270°. C. 305°. C 4.5.5.1.4.a.1 I08 AIR,RTC,LTA (Refer to figure 19.) If the airplane continues to fly on the magnetic heading as illustrated, what magnetic bearing FROM the station would be intercepted at a 30°angle? A. 090°. B. 270°. C. 310°. C 4.5.5.1.5.a.1 I08 AIR,RTC,LTA The relative bearing on an ADF changes from 265° to 260° in 2 minutes of elapsed time. If the groundspeed is 145 knots, the distance to that station would be A. 26 NM. B. 37 NM. C. 58 NM. C 4.5.5.1.6.a.1 I08 AIR,RTC The ADF indicates a wingtip bearing change of 10° in 2 minutes of elapsed time, and the TAS is 160 knots. What is the distance to the station? A. 15 NM. B. 32 NM. C. 36 NM. B 4.5.5.1.7.a.1 I08 AIR,RTC With a TAS of 115 knots, the relative bearing on an ADF changes from 090° to 095° in 1.5 minutes of elapsed time. The distance to the station would be A. 12.5 NM. B. 24.5 NM. C. 34.5 NM. C 4.5.5.1.8.a.1 I08 AIR,RTC GIVEN: Wingtip bearing change...................................5° Time elapsed between bearing change.........5 min True airspeed............................................115 kts The distance to the station is A. 36 NM. B. 57.5 NM. C. 115 NM. C 4.5.5.1.9.a.1 I08 AIR,RTC The ADF is tuned to a nondirectional radiobeacon and the relative bearing changes from 095° to 100° in 1.5 minutes of elapsed time. The time en route to that station would be A. 18 minutes. B. 24 minutes. C. 30 minutes. A 4.5.5.2.0.a.1 I08 AIR,RTC The ADF is tuned to a nondirectional radiobeacon and the relative bearing changes from 270° to 265° in 2.5 minutes of elapsed time. The time en route to that beacon would be A. 9 minutes. B. 18 minutes. C. 30 minutes. C 4.5.5.2.1.a.1 I08 AIR,RTC The ADF is tuned to a nondirectional radiobeacon and the relative bearing changes from 085° to 090° in 2 minutes of elapsed time. The time en route to the station would be A. 15 minutes. B. 18 minutes. C. 24 minutes. C 4.5.5.2.2.a.1 I08 AIR,RTC If the relative bearing changes from 090° to 100° in 2.5 minutes of elapsed time, the time en route to the station would be A. 12 minutes. B. 15 minutes. C. 18 minutes. B 4.5.5.2.3.a.1 I08 AIR,RTC The ADF is tuned to a nondirectional radiobeacon and the relative bearing changes from 090° to 100° in 2.5 minutes of elapsed time. If the true airspeed is 90 knots, the distance and time en route to that radiobeacon would be A. 15 miles and 22.5 minutes. B. 22.5 miles and 15 minutes. C. 32 miles and 18 minutes. B 4.5.5.2.8.a.1 I08 AIR,RTC While maintaining a constant heading, a relative bearing of 15° doubles in 6 minutes. The time to the station being used is A. 3 minutes. B. 6 minutes. C. 12 minutes. B 4.5.5.2.9.a.1 I08 AIR,RTC While maintaining a constant heading, the ADF needle increases from a relative bearing of 045° to 090° in 5 minutes. The time to the station being used is A. 5 minutes. B. 10 minutes. C. 15 minutes. A 4.5.5.3.0.a.1 I08 AIR,RTC While cruising at 135 knots and on a constant heading, the ADF needle decreases from a relative bearing of 315° to 270° in 7 minutes. The approximate time and distance to the station being used is A. 7 minutes and 16 miles. B. 14 minutes and 28 miles. C. 19 minutes and 38 miles. A 4.5.5.3.1.a.1 I08 AIR,RTC While maintaining a constant heading, a relative bearing of 10° doubles in 5 minutes. If the true airspeed is 105 knots, the time and distance to the station being used is approximately A. 5 minutes and 8.7 miles. B. 10 minutes and 17 miles. C. 15 minutes and 31.2 miles. A 4.5.5.3.2.a.1 I08 ALL When checking the course sensitivity of a VOR receiver, how many degrees should the OBS be rotated to move the CDI from the center to the last dot on either side? A. 5° to 10°. B. 10° to 12°. C. 18° to 20°. B 4.5.5.3.3.a.1 I08 ALL An aircraft 60 miles from a VOR station has a CDI indication of one-fifth deflection, this represents a course centerline deviation of approximately A. 6 miles. B. 2 miles. C. 1 mile. B 4.5.5.3.4.a.1 I08 ALL (Refer to figure 20.) Using instrument group 3, if the aircraft makes a 180° turn to the left and continues straight ahead, it will intercept which radial? A. 135 radial. B. 270 radial. C. 360 radial. A 4.5.5.3.5.a.1 I08 ALL (Refer to figure 20.) Which instrument shows the aircraft in a position where a 180° turn would result in the aircraft intercepting the 150 radial at a 30° angle? A. 2. B. 3. C. 4. C 4.5.5.3.6.a.1 I08 ALL (Refer to figure 20.) Which instrument shows the aircraft in a position where a straight course after a 90° left turn would result in intercepting the 180 radial? A. 2. B. 3. C. 4. B 4.5.5.3.7.a.1 I08 ALL (Refer to figure 20.) Which instrument shows the aircraft to be northwest of the VORTAC? A. 1. B. 2. C. 3. B 4.5.5.3.8.a.1 I08 ALL (Refer to figure 20.) Which instrument(s) show(s) that the aircraft is getting further from the selected VORTAC? A. 4. B. 1 and 4. C. 2 and 3. A 4.5.5.3.9.a.1 I08 AIR,RTC,LTA While maintaining a magnetic heading of 270° and a true airspeed of 120 knots, the 360 radial of a VOR is crossed at 1237 and the 350 radial is crossed at 1244. The approximate time and distance to this station are A. 42 minutes and 84 NM. B. 42 minutes and 91 NM. C. 44 minutes and 96 NM. A 4.5.5.4.0.a.1 I08 AIR,RTC,LTA (Refer to figure 21.) If the time flown between aircraft positions 2 and 3 is 13 minutes, what is the estimated time to the station? A. 13 minutes. B. 17 minutes. C. 26 minutes. A 4.5.5.4.1.a.1 I08 AIR,RTC,LTA (Refer to figure 22.) If the time flown between aircraft positions 2 and 3 is 8 minutes, what is the estimated time to the station? A. 8 minutes. B. 16 minutes. C. 48 minutes. A 4.5.5.4.2.a.1 I08 AIR,RTC,LTA (Refer to figure 23.) If the time flown between aircraft positions 2 and 3 is 13 minutes, what is the estimated time to the station? A. 7.8 minutes. B. 13 minutes. C. 26 minutes. B 4.5.5.4.3.a.1 I08 AIR,RTC,LTA (Refer to figure 24.) If the time flown between aircraft positions 2 and 3 is 15 minutes, what is the estimated time to the station? A. 15 minutes. B. 30 minutes. C. 60 minutes. A 4.5.5.4.4.a.1 I08 AIR,RTC,LTA Inbound on the 040 radial, a pilot selects the 055 radial, turns 15° to the left, and notes the time. While maintaining a constant heading, the pilot notes the time for the CDI to center is 15 minutes. Based on this information, the ETE to the station is A. 8 minutes. B. 15 minutes. C. 30 minutes. B 4.5.5.4.5.a.1 I08 AIR,RTC,LTA Inbound on the 090 radial, a pilot rotates the OBS 010° to the left, turns 010° to the right, and notes the time. While maintaining a constant heading, the pilot determines that the elapsed time for the CDI to center is 8 minutes. Based on this information, the ETE to the station is A. 8 minutes. B. 16 minutes. C. 24 minutes. A 4.5.5.4.6.a.1 I08 AIR,RTC,LTA Inbound on the 315 radial, a pilot selects the 320 radial, turns 5° to the left, and notes the time. While maintaining a constant heading, the pilot notes the time for the CDI to center is 12 minutes. The ETE to the station is A. 10 minutes. B. 12 minutes. C. 24 minutes. B 4.5.5.4.7.a.1 I08 AIR,RTC,LTA Inbound on the 190 radial, a pilot selects the 195 radial, turns 5° to the left, and notes the time. While maintaining a constant heading, the pilot notes the time for the CDI to center is 10 minutes. The ETE to the station is A. 10 minutes. B. 15 minutes. C. 20 minutes. A 4.5.5.4.8.a.1 I10 AIR,RTC,LTA (Refer to figures 25 and 25A.) During the ILS RWY 13L procedure at DSM, what altitude minimum applies if the glide slope becomes inoperative? A. 1,420 feet. B. 1,360 feet. C. 1,121 feet. B 4.5.5.4.9.a.1 I10 AIR,RTC,LTA What does the absence of the procedure turn barb on the plan view on an approach chart indicate? A. A procedure turn is not authorized. B. Teardrop-type procedure turn is authorized. C. Racetrack-type procedure turn is authorized. A 4.5.5.5.0.a.1 I10 AIR,RTC,LTA When making an instrument approach at the selected alternate airport, what landing minimums apply? A. Standard alternate minimums. B. The IFR alternate minimums listed for that airport. C. The landing minimums published for the type of procedure selected. C 4.5.5.5.1.a.1 J01 ALL How should the pilot make a VOR receiver check when the aircraft is located on the designated checkpoint on the airport surface? A. Set the OBS on 180° plus or minus 4°; the CDI should center with a FROM indication. B. Set the OBS on the designated radial. The CDI must center within plus or minus 4° of that radial with a FROM indication. C. With the aircraft headed directly toward the VOR and the OBS set to 000°, the CDI should center within plus or minus 4° of that radial with a TO indication. B 4.5.5.5.2.a.1 J01 ALL When using VOT to make a VOR receiver check, the CDI should be centered and the OBS should indicate that the aircraft is on the A. 090 radial. B. 180 radial. C. 360 radial. C 4.5.5.5.3.a.1 J01 ALL When the CDI needle is centered during an airborne VOR check, the omnibearing selector and the TO/FROM indicator should read A. within 4° of the selected radial. B. within 6° of the selected radial. C. 0° TO, only if you are due south of the VOR. B 4.5.5.5.6.a.1 J16 AIR,RTC,LTA Which is true regarding the use of a Standard Instrument Departure (SID) chart? A. At airfields where SID's have been established, SID usage is mandatory for IFR departures. B. To use a SID, the pilot must possess at least the textual description of the approved standard departure. C. To use a SID, the pilot must possess both the textual and graphic form of the approved standard departure. B 4.5.5.5.7.a.1 J17 AIR,RTC,LTA Which is true regarding STAR's? STAR's are A. used to separate IFR and VFR traffic. B. established to simplify clearance delivery procedures. C. used at certain airports to decrease traffic congestion. B 4.5.5.5.8.a.1 J18 AIR,RTC,LTA While being radar vectored, an approach clearance is received. The last assigned altitude should be maintained until A. reaching the FAF. B. advised to begin descent. C. established on a segment of a published route or instrument approach procedure. C 4.5.5.5.9.a.1 J25 ALL Flight Service Stations in the conterminous 48 United States having voice capability on VOR's or radiobeacons (NDB's) broadcast A. AIRMET's and SIGMET's at 15 minutes past the hour and each 15 minutes thereafter as long as they are in effect. B. AIRMET's and Nonconvective SIGMET's upon receipt and at 15 minutes and 45 minutes past the hour for the first hour after issuance. C. hourly weather reports at 15 and 45 minutes past each hour for those reporting stations within approximately 150 NM of the broadcast stations. B 4.5.5.6.0.a.1 J25 ALL To obtain a continuous transcribed weather briefing including winds aloft and route forecasts for a cross- country flight, a pilot could monitor A. a TWEB on a low-frequency radio receiver. B. the regularly scheduled weather broadcast on a VOR frequency. C. a high-frequency radio receiver tuned to En Route Flight Advisory Service. A 4.5.5.6.1.a.1 J33 AIR,RTC,LTA (Refer to figures 26 and 26A.) The final approach fix for the precision approach is located at A. DENAY Intersection. B. Glide slope intercept. C. ROMEN Intersection/Locator outer marker. B 4.5.5.6.4.a.1 J37 ALL Which is true relating to the blue and magenta colors used to depict airports on Sectional Aeronautical Charts? A. Class E airports are shown in blue; Class C and D are magenta. B. Class B airports are shown in blue; Class D and E are magenta. C. Class E airports are shown in magenta; Class B, C, and D are blue. C 4.5.5.6.5.a.1 J37 ALL (Refer to figure 52, point A.) The floor of the Class E airspace above Georgetown Airport (Q61) is at A. the surface. B. 3,788 feet MSL. C. 700 feet AGL. B 4.5.5.6.6.a.1 J37 ALL (Refer to figure 52, point G.) The floor of Class E airspace over the town of Woodland is A. 700 feet AGL over part of the town and no floor over the remainder. B. 1,200 feet AGL over part of the town and no floor over the remainder. C. both 700 feet and 1,200 feet AGL. C 4.5.5.6.7.a.1 J37 ALL (Refer to figure 52, point E.) The floor of the Class E airspace over University Airport (0O5) is A. the surface. B. 700 feet AGL. C. 1,200 feet AGL. B 4.5.5.6.8.a.1 J37 ALL (Refer to figure 52, point H.) The floor of the Class E airspace over the town of Auburn is A. 1,200 feet MSL. B. 700 feet AGL. C. 1,200 feet AGL. C 4.5.5.6.9.a.1 J37 ALL (Refer to figure 53, point A.) This thin black shaded line is most likely A. an arrival route. B. a military training route. C. a state boundary line. B 4.5.5.7.0.a.1 J37 ALL (Refer to figure 53, point B.) The 16 indicates A. an antenna top at 1,600 feet AGL. B. the maximum elevation figure for that quadrangle. C. the minimum safe sector altitude for that quadrangle. B 4.5.5.7.2.a.1 J37 ALL (Refer to figure 54, point A.) What minimum altitude is required to avoid the Livermore Airport (LVK) Class D airspace? A. 2,503 feet MSL. B. 2,901 feet MSL. C. 3,297 feet MSL. B 4.5.5.7.4.a.1 J37 ALL (Refer to figure 54, point A.) Flight over Livermore Airport (LVK) at 3000 feet MSL A. requires a transponder, but ATC communication is not necessary. B. does not require a transponder or ATC communication. C. cannot be accomplished without meeting all Class B airspace requirements. X/A 4.5.5.7.6.a.1 J37 ALL (Refer to figure 54, point D.) The thinner outer blue circle depicted around San Francisco International Airport is A. the outer segment of Class B airspace. B. an area within which an appropriate transponder must be used from outside of the Class B airspace from the surface to 10,000 feet MSL. C. a Mode C veil boundary where an aircraft may penetrate without a transponder provided it remains below 8,000 feet. B 4.5.5.7.7.a.1 J37 ALL When fixed wing Special Visual Flight Rules (SVFR) operation is prohibited at an airport, the sectional aeronautical chart will A. depict ``TTTT'' symbols in a circular fashion around that airport. B. State ``No SVFR'' near the airport symbol. C. not depict this information. B 4.5.5.8.1.a.1 J37 ALL (Refer to figure 52, point D.) The highest obstruction with high intensity lighting within 10 NM of Lincoln Airport (O51) is how high above the ground? A. 1,254 feet. B. 662 feet. C. 299 feet. C 4.5.5.8.3.a.1 J37 ALL (Refer to figure 52, point F.) Mosier Airport is A. an airport restricted to use by private and recreational pilots. B. a restricted military stage field within restricted airspace. C. a nonpublic use airport. C 4.5.5.8.4.a.1 J37 ALL (Refer to figure 54, point B.) After departing from Rio Vista Airport (O88) with a southerly wind, you discover flight visibility to be approximately 2 1/2 miles, you must A. contact Travis AFB on remote frequency 122.8 to advise of your intentions. B. stay below 1,200 feet to remain in Class G. C. stay below 700 feet to remain in Class G. C 4.5.5.8.5.a.1 J37 ALL (Refer to figure 52, point D.) The terrain at the obstruction approximately 8 NM east southeast of the Lincoln Airport is approximately how much higher than the airport elevation? A. 376 feet. B. 835 feet. C. 1,135 feet. B 4.5.5.8.7.a.1 J37 ALL (Refer to figure 54, point F.) The Class C airspace at Metropolitan Oakland International (OAK) which extends from the surface upward has a ceiling of A. both 2,100 feet and 3,000 feet MSL. B. 8,000 feet MSL. C. 2,100 feet AGL. A 4.5.5.8.8.a.1 J37 ALL (Refer to figure 53.) GIVEN: Altitude.................................................1,000 ft AGL Position...................................7 NM north of point E Time.......................................................3 p.m. local Flight visibility..................................................1 SM You are VFR approaching Madera Airport (point E) for a landing from the north. You A. are in violation of the FAR's; you need 3 miles of visibility under VFR. B. are required to descend to below 700 feet AGL before entering Class E airspace and may continue for landing. C. may descend to 800 feet AGL (Pattern Altitude) after entering Class E airspace and continue to the airport. B 4.5.5.9.1.a.1 J37 AIR,RTC,LTA (Refer to figures 55 and 55a.) En route on V112 from BTG VORTAC to LTJ VORTAC, the minimum altitude crossing GYMME intersection is A. 6,400 feet. B. 6,500 feet. C. 7,000 feet. C 4.5.5.9.2.a.1 J37 AIR,RTC,LTA (Refer to figures 55 and 55a.) En route on V448 from YKM VORTAC to BTG VORTAC, what minimum navigation equipment is required to identify ANGOO intersection? A. One VOR receiver. B. One VOR receiver and DME. C. Two VOR receivers. A 4.5.5.9.3.a.1 J37 AIR,RTC,LTA (Refer to figures 55 and 55a.) En route on V468 from BTG VORTAC to YKM VORTAC, the minimum en route altitude at TROTS intersection is A. 7,100 feet. B. 10,000 feet. C. 11,500 feet. C 4.5.5.9.4.a.1 J42 AIR,RTC,LTA (Refer to figures 27 and 27a.) In the DEN ILS RWY 35R procedure the FAF intercept altitude is A. 7,488 feet MSL. B. 7,500 feet MSL. C. 9,000 feet MSL. B 4.5.5.9.5.a.1 J42 AIR,RTC,LTA (Refer to figures 27 and 27a.) The symbol [8100] in the MSA circle of the ILS RWY 35R procedure at DEN represents a minimum safe sector altitude within 25 NM of A. Denver VORTAC. B. Gandi outer marker. C. Denver/Stapleton International Airport. A 4.5.5.9.6.a.1 J42 AIR,RTC,LTA (Refer to figures 28 and 28a.) During the ILS RWY 31R procedure at DSM, the minimum altitude for glide slope interception is A. 2,365 feet MSL. B. 2,500 feet MSL. C. 3,000 feet MSL. X/B 4.5.5.9.7.a.1 J42 AIR,RTC,LTA (Refer to figures 28 and 28a.) If the glide slope becomes inoperative during the ILS RWY 31R procedure at DSM, what MDA applies? A. 1,157 feet. B. 1,320 feet. C. 1,360 feet. B 4.5.5.9.8.a.1 J42 AIR,RTC,LTA (Refer to figures 29 and 29a.) When approaching the ATL ILS RWY 8L, how far from the FAF is the missed approach point? A. 4.8 NM. B. 5.2 NM. C. 12.0 NM. B 4.5.5.9.9.a.1 J42 AIR,RTC,LTA (Refer to figures 30 and 30a.) When approaching the VOR/DME-A, the symbol [2800] in the MSA circle represents a minimum safe sector altitude within 25 NM of A. DEANI intersection. B. White Cloud VORTAC. C. Baldwin Municipal Airport. B 4.5.6.0.0.a.1 B07 AIR,RTC,LTA (Refer to figures 30 and 30a.) What minimum navigation equipment is required to complete the VOR/DME-A procedure? A. One VOR receiver. B. One VOR receiver and DME. C. Two VOR receivers and DME. B 5.5.6.0.1.a.1 B08 ALL During a night operation, the pilot of aircraft 1 sees only the green light of aircraft 2. If the aircraft are converging, which pilot has the right-of-way? The pilot of aircraft A. 2; aircraft 2 is to the right of aircraft 1. B. 1; aircraft 1 is to the right of aircraft 2. C. 2; aircraft 2 is to the left of aircraft 1. B 5.5.6.0.4.a.1 H01 ALL Why should flight speeds above VNE be avoided? A. Excessive induced drag will result in structural failure. B. Design limit load factors may be exceeded, if gusts are encountered. C. Control effectiveness is so impaired that the aircraft becomes uncontrollable. X/B 5.5.6.0.5.a.1 H01 ALL Maximum structural cruising speed is the maximum speed at which an airplane can be operated during A. abrupt maneuvers. B. normal operations. C. flight in smooth air. B 5.5.6.0.6.a.1 H02 AIR,RTC,LTA Applying carburetor heat will A. not affect the mixture. B. lean the fuel/air mixture. C. enrich the fuel/air mixture. C 5.5.6.0.7.a.1 H02 AIR,RTC,LTA An abnormally high engine oil temperature indication may be caused by A. a defective bearing. B. the oil level being too low. C. operating with an excessively rich mixture. B 5.5.6.0.8.a.1 H02 AIR,RTC,LTA What will occur if no leaning is made with the mixture control as the flight altitude increases? A. The volume of air entering the carburetor decreases and the amount of fuel decreases. B. The density of air entering the carburetor decreases and the amount of fuel increases. C. The density of air entering the carburetor decreases and the amount of fuel remains constant. C 5.5.6.0.9.a.1 H02 AIR,RTC,LTA Unless adjusted, the fuel/air mixture becomes richer with an increase in altitude because the amount of fuel A. decreases while the volume of air decreases. B. remains constant while the volume of air decreases. C. remains constant while the density of air decreases. C 5.5.6.1.0.a.1 H02 AIR,RTC,LTA The basic purpose of adjusting the fuel/air mixture control at altitude is to A. decrease the fuel flow to compensate for decreased air density. B. decrease the amount of fuel in the mixture to compensate for increased air density. C. increase the amount of fuel in the mixture to compensate for the decrease in pressure and density of the air. A 5.5.6.1.1.a.1 H02 AIR,RTC,LTA At high altitudes, an excessively rich mixture will cause the A. engine to overheat. B. fouling of spark plugs. C. engine to operate smoother even though fuel consumption is increased. B 5.5.6.1.6.a.1 H04 AIR,RTC,GLI (Refer to figure 31.) If the tower-reported surface wind is 010° at 18 knots, what is the crosswind component for a Rwy 08 landing? A. 7 knots. B. 15 knots. C. 17 knots. C 5.5.6.1.7.a.1 H04 AIR,RTC,GLI (Refer to figure 31.) The surface wind is 180° at 25 knots. What is the crosswind component for a Rwy 13 landing? A. 19 knots. B. 21 knots. C. 23 knots. A 5.5.6.1.8.a.1 H04 AIR,RTC,GLI (Refer to figure 31.) What is the headwind component for a Rwy 13 takeoff if the surface wind is 190° at 15 knots? A. 7 knots. B. 13 knots. C. 15 knots. A 5.5.6.3.2.a.1 H11 AIR,RTC When computing weight and balance, the empty weight includes the weight of the airframe, engine(s), and all items of operating equipment permanently installed. Empty weight also includes A. the unusable fuel, hydraulic fluid, and undrainable oil or, in some aircraft, all of the oil. B. all usable fuel, maximum oil, hydraulic fluid, but does not include the weight of pilot, passengers, or baggage. C. all usable fuel and oil, but does not include any radio equipment or instruments that were installed by someone other than the manufacturer. A 5.5.6.3.4.a.1 H12 ALL The CG of an aircraft can be determined by which of the following methods? A. Dividing total arms by total moments. B. Multiplying total arms by total weight. C. Dividing total moments by total weight. C 5.5.6.3.5.a.1 H12 ALL The CG of an aircraft may be determined by A. dividing total arms by total moments. B. dividing total moments by total weight. C. multiplying total weight by total moments. B 5.5.6.3.6.a.1 H12 ALL GIVEN: Weight A - 155 pounds at 45 inches aft of datum Weight B - 165 pounds at 145 inches aft of datum Weight C - 95 pounds at 185 inches aft of datum Based on this information, where would the CG be located aft of datum? A. 86.0 inches. B. 116.8 inches. C. 125.0 inches. B 5.5.6.3.7.a.1 H12 ALL GIVEN: Weight A - 140 pounds at 17 inches aft of datum Weight B - 120 pounds at 110 inches aft of datum Weight C - 85 pounds at 210 inches aft of datum Based on this information, the CG would be located how far aft of datum? A. 89.11 inches. B. 96.89 inches. C. 106.92 inches. B 5.5.6.3.8.a.1 H12 ALL GIVEN: Weight A - 135 pounds at 15 inches aft of datum Weight B - 205 pounds at 117 inches aft of datum Weight C - 85 pounds at 195 inches aft of datum Based on this information, the CG would be located how far aft of datum? A. 100.2 inches. B. 109.0 inches. C. 121.7 inches. A 5.5.6.3.9.a.1 H12 ALL GIVEN: Weight A - 175 pounds at 135 inches aft of datum Weight B - 135 pounds at 115 inches aft of datum Weight C - 75 pounds at 85 inches aft of datum The CG for the combined weights would be located how far aft of datum? A. 91.76 inches. B. 111.67 inches. C. 118.24 inches. C 5.5.6.4.4.a.1 H12 RTC (Refer to figure 37.) GIVEN: WEIGHT MOMENT Gyroplane basic weight 1,315 150.1 (oil included) Pilot weight 140 ? Passenger weight 150 ? 27 gal fuel 162 ? The CG is located A. outside the CG envelope; the maximum gross weight is exceeded. B. outside the CG envelope; the maximum gross weight and the gross-weight moment are exceeded. C. within the CG envelope; neither maximum gross weight nor gross-weight moment is exceeded. C 5.5.6.4.5.a.1 H12 RTC (Refer to figure 37.) GIVEN: WEIGHT MOMENT Gyroplane basic weight 1,315 154.0 (oil included) Pilot weight 145 ? Passenger weight 153 ? 27 gal fuel 162 ? The CG is located A. outside the CG envelope; the maximum gross weight is exceeded. B. outside the CG envelope; but the maximum gross weight is not exceeded. C. within the CG envelope; neither maximum gross weight nor gross-weight moment is exceeded. B 5.5.6.5.3.a.1 H50 AIR,RTC Frequent inspections should be made of aircraft exhaust manifold-type heating systems to minimize the possibility of A. exhaust gases leaking into the cockpit. B. a power loss due to back pressure in the exhaust system. C. a cold-running engine due to the heat withdrawn by the heater. A 5.5.6.5.7.a.1 H56 AIR,RTC (Refer to figure 51.) The pilot generally calls ground control after landing when the aircraft is completely clear of the runway. This is when the aircraft A. passes the red symbol shown at the top of the figure. B. is on the dashed-line side of the middle symbol. C. is on the solid-line side of the middle symbol. C 5.5.6.5.8.a.1 H56 AIR,RTC (Refer to figure 51.) The red symbol at the top would most likely be found A. upon exiting all runways prior to calling ground control. B. where a roadway may be mistaken as a taxiway. C. near the approach end of ILS runways. B 5.5.6.5.9.a.1 H56 AIR,RTC (Refer to figure 51.) While clearing an active runway you are most likely clear of the ILS critical area when you pass which symbol? A. Top red. B. Middle yellow. C. Bottom yellow. C 5.5.6.6.0.a.1 H56 AIR,RTC (Refer to figure 51.) Which symbol does not directly address runway incursion with other aircraft? A. Top red. B. Middle yellow. C. Bottom yellow. A 5.5.6.7.1.a.1 H71 RTC During the full flare portion of a power-off landing, the rotor RPM tends to A. remain constant. B. increase initially. C. decrease initially. B 5.5.6.7.2.a.1 H71 RTC Which would produce the slowest rotor RPM? A. A vertical descent with power. B. A vertical descent without power. C. Pushing over after a steep climb. C 5.5.6.7.3.a.1 H73 RTC If the RPM is low and the manifold pressure is high, what initial corrective action should be taken? A. Increase the throttle. B. Lower the collective pitch. C. Raise the collective pitch. B 5.5.6.7.4.a.1 H73 RTC During climbing flight, the manifold pressure is low and the RPM is high. What initial corrective action should be taken? A. Increase the throttle. B. Decrease the throttle. C. Raise the collective pitch. C 5.5.6.7.5.a.1 H73 RTC During level flight, if the manifold pressure is high and the RPM is low, what initial corrective action should be made? A. Decrease the throttle. B. Increase the throttle. C. Lower the collective pitch. C 5.5.6.7.6.a.1 H75 RTC When operating a helicopter in conditions favorable for carburetor icing, the carburetor heat should be A. adjusted to keep the carburetor air temperature gauge indicating in the green arc at all times. B. OFF for takeoffs, adjusted to keep the carburetor air temperature gauge indicating in the green arc at all other times. C. OFF during takeoffs, approaches, and landings; adjusted to keep the carburetor air temperature gauge indicating in the green arc at all other times. B 5.5.6.7.7.a.1 H76 RTC (Refer to figure 39.) GIVEN: MOMENT WEIGHT ARM (IN) (IN.-LBS) Empty weight 1,700 +6.0 +10,200 Pilot weight 200 -31.0 ? Oil (8 qt, ? +1.0 ? all usable) Fuel (50 gal, ? +2.0 ? all usable) Baggage 30 -31.0 ? TOTALS ? ? ? If the datum line is located at station 0, the CG is located approximately A. 1.64 inches aft of datum. B. 1.64 inches forward of datum. C. 1.66 inches forward of datum. A 5.5.6.7.8.a.1 H76 RTC (Refer to figure 40.) GIVEN: Basic weight (oil is included)..............830 lb Basic weight moment (1,000/in.-lb).....104.8 Pilot weight........................................175 lb Passenger weight ................................160 lb Fuel..................................................19.2 gal The CG is located A. well aft of the aft CG limit. B. within the CG envelope. C. forward of the forward CG limit. A 5.5.6.7.9.a.1 H76 RTC GIVEN: LNG. LNG. LAT. LAT. WT ARM MOM. ARM. MOM. Empty weight 1700 116.1 ? +0.2 -- Fuel (75 gal ? 110.0 ? -- -- at 6.8 ppg) Oil 12 179.0 ? -- -- Pilot (right seat) 175 65.0 ? +12.5 ? Passenger 195 104.0 ? -13.3 ? (left seat) TOTALS ? ? ? ? ? Determine the longitudinal and lateral CG respectively. A. 109.35" and -.04". B. 110.43" and +.02". C. 110.83" and -.02". C 5.5.6.8.0.a.1 H76 RTC A helicopter is loaded in such a manner that the CG is located aft of the aft allowable CG limit. Which is true about this situation? A. In case of an autorotation, sufficient aft cyclic control may not be available to flare properly. B. This condition would become more hazardous as fuel is consumed, if the main fuel tank is located aft of the rotor mast. C. If the helicopter should pitchup due to gusty winds during high-speed flight, there may not be sufficient forward cyclic control available to lower the nose. C 5.5.6.8.1.a.1 H76 RTC A helicopter is loaded in such a manner that the CG is located forward of the allowable CG limit. Which is true about this situation? A. This condition would become less hazardous as fuel is consumed if the fuel tank is located aft of the rotor mast. B. In case of engine failure and the resulting autorotation, sufficient cyclic control may not be available to flare properly to land. C. Should the aircraft pitchup during cruise flight due to gusty winds, there may not be enough forward cyclic control available to lower the nose. B 5.5.6.8.2.a.1 H76 ALL With respect to using the weight information given in a typical aircraft owner's manual for computing gross weight, it is important to know that if items have been installed in the aircraft in addition to the original equipment, the A. allowable useful load is decreased. B. allowable useful load remains unchanged. C. maximum allowable gross weight is increased. A 5.5.6.8.3.a.1 H77 RTC (Refer to figure 41.) GIVEN: Helicopter gross weight...................1,225 lb Ambient temperature...........................77 °F Determine the in-ground-effect hover ceiling. A. 6,750 feet. B. 7,250 feet. C. 8,000 feet. A 5.5.6.8.4.a.1 H77 RTC (Refer to figure 41.) GIVEN: Helicopter gross weight...................1,175 lb Ambient temperature..........................95 °F Determine the out-of-ground effect hover ceiling. A. 5,000 feet. B. 5,250 feet. C. 6,250 feet. B 5.5.6.8.5.a.1 H77 RTC (Refer to figure 41.) GIVEN: Helicopter gross weight.................1,275 lb Ambient temperature...........................9 °F Determine the in ground effect hover ceiling. A. 6,600 feet. B. 7,900 feet. C. 8,750 feet. B 5.5.6.8.6.a.1 H77 RTC As altitude increases, the VNE of a helicopter will A. increase. B. decrease. C. remain the same. B 5.5.6.8.7.a.1 H77 RTC (Refer to figure 42.) Departure is planned from a heliport that has a reported pressure altitude of 4,100 feet. What rate of climb could be expected in this helicopter if the ambient temperature is 90 °F? A. 210 ft/min. B. 250 ft/min. C. 390 ft/min. B 5.5.6.8.8.a.1 H77 RTC (Refer to figure 42.) Departure is planned for a flight from a heliport with a pressure altitude of 3,800 feet. What rate of climb could be expected in this helicopter during departure if the ambient temperature is 70 °F? A. 330 ft/min. B. 360 ft/min. C. 400 ft/min. A 5.5.6.8.9.a.1 H77 RTC (Refer to figure 43.) GIVEN: Ambient temperature........................60 °F Pressure altitude...........................2,000 ft What is the rate of climb? A. 480 ft/min. B. 515 ft/min. C. 540 ft/min. B 5.5.6.9.0.a.1 H77 RTC (Refer to figure 43.) GIVEN: Ambient temperature........................80 °F Pressure altitude...........................2,500 ft What is the rate of climb? A. 350 ft/min. B. 395 ft/min. C. 420 ft/min. B 5.5.6.9.1.a.1 H77 RTC (Refer to figure 44.) GIVEN: Ambient temperature.........................40 °F Pressure altitude............................1,000 ft What is the rate of climb? A. 810 ft/min. B. 830 ft/min. C. 860 ft/min. C 5.5.6.9.2.a.1 H77 RTC (Refer to figure 44.) GIVEN: Ambient temperature..........................60 °F Pressure altitude.............................2,000 ft What is the rate of climb? A. 705 ft/min. B. 630 ft/min. C. 755 ft/min. A 5.5.6.9.3.a.1 H77 RTC (Refer to figures 45 and 46.) GIVEN: Pressure altitude.............................4,000 ft Ambient temperature.........................80 °F To clear a 50-foot obstacle, a jump takeoff would require A. more distance than a running takeoff. B. less distance than a running takeoff. C. the same distance as a running takeoff. A 5.5.6.9.4.a.1 H77 RTC (Refer to figures 45 and 46.) GIVEN: Pressure altitude.............................4,000 ft Ambient temperature.........................80 °F The takeoff distance to clear a 50-foot obstacle is A. 1,225 feet for a jump takeoff. B. 1,440 feet for a running takeoff. C. less for a running takeoff than for a jump takeoff. C 5.5.6.9.5.a.1 H78 RTC The antitorque system fails during cruising flight and a powered approach landing is commenced. If the helicopter yaws to the right just prior to touchdown, what could the pilot do to help swing the nose to the left? A. Increase the throttle. B. Decrease the throttle. C. Increase collective pitch. B 5.5.6.9.6.a.1 H78 RTC If antitorque failure occurred during cruising flight, what could be done to help straighten out a left yaw prior to touchdown? A. A normal running landing should be made. B. Make a running landing using partial power and left cyclic. C. Apply available throttle to help swing the nose to the right just prior to touchdown. C 5.5.6.9.7.a.1 H78 RTC Should a helicopter pilot ever be concerned about ground resonance during takeoff? A. No; ground resonance occurs only during an autorotative touchdown. B. Yes; although it is more likely to occur on landing, it can occur during takeoff. C. Yes, but only during slope takeoffs. B 5.5.6.9.8.a.1 H78 RTC An excessively steep approach angle and abnormally slow closure rate should be avoided during an approach to a hover, primarily because A. the airspeed indicator would be unreliable. B. a go-around would be very difficult to accomplish. C. settling with power could develop, particularly during the termination. C 5.5.6.9.9.a.1 H78 RTC During a near-vertical power approach into a confined area with the airspeed near zero, what hazardous condition may develop? A. Ground resonance. B. Settling with power. C. Blade stall vibration. B 5.5.7.0.0.a.1 H78 RTC Which procedure will result in recovery from settling with power? A. Increase collective pitch and power. B. Maintain constant collective pitch and increase throttle. C. Increase forward speed and partially lower collective pitch. C 5.5.7.0.1.a.1 H78 RTC The addition of power in a settling with power situation produces an A. increase in airspeed. B. even greater rate of descent. C. increase in cyclic control effectiveness. B 5.5.7.0.2.a.1 H78 RTC Under which situation is accidental settling with power likely to occur? A. A steep approach in which the airspeed is permitted to drop to nearly zero. B. A shallow approach in which the airspeed is permitted to drop below 10 MPH. C. Hovering in ground effect during calm wind, high-density altitude conditions. A 5.5.7.0.3.a.1 H78 RTC Which is true with respect to recovering from an accidental settling with power situation? A. Antitorque pedals should not be utilized during the recovery. B. Recovery can be accomplished by increasing rotor RPM, reducing forward airspeed, and minimizing maneuvering. C. Since the inboard portions of the main rotor blades are stalled, cyclic control effectiveness will be reduced during the initial portion of the recovery. C 5.5.7.0.4.a.1 H78 RTC When operating at high forward airspeed, retreating blade stall is more likely to occur under conditions of A. low gross weight, high density altitude, and smooth air. B. high gross weight, low density altitude, and smooth air. C. high gross weight, high density altitude, and turbulent air. C 5.5.7.0.5.a.1 H78 RTC What are the major indications of an incipient retreating blade stall situation, in order of occurrence? A. Low-frequency vibration, pitchup of the nose, and a tendency for the helicopter to roll. B. Slow pitchup of the nose, high-frequency vibration, and a tendency for the helicopter to roll. C. Slow pitchup of the nose, tendency for the helicopter to roll, followed by a medium-frequency vibration. A 5.5.7.0.6.a.1 H78 RTC How should a pilot react at the onset of retreating blade stall? A. Reduce collective pitch, rotor RPM, and forward airspeed. B. Reduce collective pitch, increase rotor RPM, and reduce forward airspeed. C. Increase collective pitch, reduce rotor RPM, and reduce forward airspeed. B 5.5.7.0.7.a.1 H79 RTC The most power will be required to hover over which surface? A. High grass. B. Concrete ramp. C. Rough/uneven ground. A 5.5.7.0.8.a.1 H79 RTC Which flight technique is recommended for use during hot weather? A. During takeoff, accelerate quickly into forward flight. B. During takeoff, accelerate slowly into forward flight. C. Use minimum allowable RPM and maximum allowable manifold pressure during all phases of flight. B 5.5.7.0.9.a.1 H80 RTC To taxi on the surface in a safe and efficient manner, helicopter pilots should use the A. cyclic pitch to control starting, taxi speed, and stopping. B. collective pitch to control starting, taxi speed, and stopping. C. antitorque pedals to correct for drift during crosswind conditions. B 5.5.7.1.0.a.1 H80 RTC During surface taxiing, the cyclic pitch stick is used to control A. heading. B. ground track. C. forward movement. B 5.5.7.1.1.a.1 H80 RTC To taxi on the surface in a safe and efficient manner, one should use the cyclic pitch to A. start and stop aircraft movement. B. maintain heading during crosswind conditions. C. correct for drift during crosswind conditions. C 5.5.7.1.2.a.1 H80 RTC A pilot is hovering during calm wind conditions. The greatest amount of engine power will be required when A. ground effect exists. B. making a left-pedal turn. C. making a right-pedal turn. B 5.5.7.1.3.a.1 H80 RTC Which statement is true about an autorotative descent? A. Generally, only the cyclic control is used to make turns. B. The pilot should use the collective pitch control to control the rate of descent. C. The rotor RPM will tend to decrease if a tight turn is made with a heavily loaded helicopter. A 5.5.7.1.4.a.1 H80 RTC Using right pedal to assist a right turn during an autorotative descent will probably result in what actions? A. A decrease in rotor RPM, pitch up of the nose, decrease in sink rate, and increase in indicated airspeed. B. An increase in rotor RPM, pitch up of the nose, decrease in sink rate, and increase in indicated airspeed. C. An increase in rotor RPM, pitch down of the nose, increase in sink rate, and decrease in indicated airspeed. C 5.5.7.1.5.a.1 H80 RTC Using left pedal to assist a left turn during an autorotative descent will probably cause the rotor RPM to A. increase and the airspeed to decrease. B. decrease and the aircraft nose to pitch down. C. increase and the aircraft nose to pitch down. C 5.5.7.1.6.a.1 H80 RTC When planning slope operations, only slopes of 5° gradient or less should be considered, primarily because A. ground effect is lost on slopes of steeper gradient. B. downwash turbulence is more severe on slopes of steeper gradient. C. most helicopters are not designed for operations on slopes of steeper gradient. C 5.5.7.1.7.a.1 H80 RTC When making a slope landing, the cyclic pitch control should be used to A. lower the downslope skid to the ground. B. hold the upslope skid against the slope. C. place the rotor disc parallel to the slope. B 5.5.7.1.8.a.1 H80 RTC Takeoff from a slope is normally accomplished by A. making a downslope running takeoff if the surface is smooth. B. simultaneously applying collective pitch and downslope cyclic control. C. bringing the helicopter to a level attitude before completely leaving the ground. C 5.5.7.1.9.a.1 H80 RTC What is the procedure for a slope landing? A. Use maximum RPM and maximum manifold pressure. B. If the slope is 10° or less, the landing should be made perpendicular to the slope. C. When parallel to the slope, slowly lower the upslope skid to the ground prior to lowering the downslope skid. C 5.5.7.2.0.a.1 H80 RTC You are hovering during calm wind conditions and decide to make a right-pedal turn. In most helicopters equipped with reciprocating engines, the engine RPM will tend to A. increase. B. decrease. C. remain unaffected. A 5.5.7.2.1.a.1 H80 RTC During calm wind conditions, in most helicopters, which of these flight operations would require the most power? A. A left-pedal turn. B. A right-pedal turn. C. Hovering in ground effect. A 5.5.7.2.2.a.1 H80 RTC If complete power failure should occur while cruising at altitude, the pilot should A. partially lower the collective pitch, close the throttle, then completely lower the collective pitch. B. lower the collective pitch as necessary to maintain proper rotor RPM, and apply right pedal to correct for yaw. C. close the throttle, lower the collective pitch to the full-down position, apply left pedal to correct for yaw, and establish a normal power-off glide. B 5.5.7.2.3.a.1 H80 RTC When making an autorotation to touchdown, what action is most appropriate? A. A slightly nose-high attitude at touchdown is the proper procedure. B. The skids should be in a longitudinally level attitude at touchdown. C. Aft cyclic application after touchdown is desirable to help decrease ground run. B 5.5.7.2.4.a.1 H80 RTC During the entry into a quick stop, how should the collective pitch control be used? It should be A. lowered as necessary to prevent ballooning. B. raised as necessary to prevent a rotor overspeed. C. raised as necessary to prevent a loss of altitude. A 5.5.7.2.5.a.1 H80 RTC During a normal approach to a hover, the collective pitch control is used primarily to A. maintain RPM. B. control the rate of closure. C. control the angle of descent. C 5.5.7.2.6.a.1 H80 RTC During a normal approach to a hover, the cyclic pitch is used primarily to A. maintain heading. B. control rate of closure. C. control angle of descent. B 5.5.7.2.7.a.1 H80 RTC Normal RPM should be maintained during a running landing primarily to ensure A. adequate directional control until the helicopter stops. B. that sufficient lift is available should an emergency develop. C. longitudinal and lateral control, especially if the helicopter is heavily loaded or high density altitude conditions exist. A 5.5.7.2.8.a.1 H80 RTC Which is true concerning a running takeoff? A. If a helicopter cannot be lifted vertically, a running takeoff should be made. B. One advantage of a running takeoff is that the additional airspeed can be converted quickly to altitude. C. A running takeoff may be possible when gross weight or density altitude prevents a sustained hover at normal hovering altitude. C 5.5.7.2.9.a.1 H81 RTC When conducting a confined area-type operation, the primary purpose of the high reconnaissance is to determine the A. type of approach to be made. B. suitability of the area for landing. C. height of the obstructions surrounding the area. B 5.5.7.3.0.a.1 H81 RTC During a pinnacle approach to a rooftop heliport under conditions of high wind and turbulence, the pilot should make a A. shallow approach, maintaining a constant line of descent with cyclic applications. B. normal approach, maintaining a slower-than-normal rate of descent with cyclic applications. C. steeper-than-normal approach, maintaining the desired angle of descent with collective applications. C 5.5.7.3.1.a.1 H81 RTC What type approach should be made to a rooftop heliport under conditions of relatively high wind and turbulence? A. A normal approach. B. A steeper-than-normal approach. C. A shallower-than-normal approach. B 5.5.7.3.2.a.1 H81 RTC If turbulence and downdrafts are expected during a pinnacle approach to a rooftop heliport, plan to make a A. steeper-than-normal approach. B. normal approach, maintaining a lower-than-normal airspeed. C. shallow approach, maintaining a higher-than-normal airspeed. A 5.5.7.3.3.a.1 H91 RTC If ground resonance is experienced during rotor spin-up, what action should you take? A. Taxi to a smooth area. B. Make a normal takeoff immediately. C. Close the throttle and slowly raise the spin-up lever. C 5.5.7.3.4.a.1 H91 RTC The principal factor limiting the never-exceed speed (VNE) of a gyroplane is A. turbulence and altitude. B. blade-tip speed, which must remain below the speed of sound. C. lack of sufficient cyclic stick control to compensate for dissymmetry of lift or retreating blade stall, depending on which occurs first. C 5.5.7.3.5.a.1 H91 RTC Why should gyroplane operations within the cross-hatched portion of a Height vs. Velocity chart be avoided? A. The rotor RPM may build excessively high if it is necessary to flare at such low altitudes. B. Sufficient airspeed may not be available to ensure a safe landing in case of an engine failure. C. Turbulence near the surface can dephase the blade dampers causing geometric unbalanced conditions on the rotor system. B 5.5.7.3.6.a.1 H91 RTC The principal reason the shaded area of a Height vs. Velocity chart should be avoided is A. rotor RPM may decay before ground contact is made if an engine failure should occur. B. rotor RPM may build excessively high if it is necessary to flare at such low altitudes. C. insufficient airspeed would be available to ensure a safe landing in case of an engine failure. C 5.5.7.3.7.a.1 H92 RTC During the transition from pre-rotation to flight, all rotor blades change pitch A. simultaneously to the same angle of incidence. B. simultaneously but to different angles of incidence. C. to the same degree at the same point in the cycle of rotation. B 5.5.7.3.8.a.1 H94 RTC Select the true statement concerning gyroplane taxi procedures. A. Avoid abrupt control movements when blades are turning. B. The cyclic stick should be held in the neutral position at all times. C. The cyclic stick should be held slightly aft of neutral at all times. A 5.5.7.4.0.a.1 I04 ALL To determine pressure altitude prior to takeoff, the altimeter should be set to A. the current altimeter setting. B. 29.92" Hg and the altimeter indication noted. C. the field elevation and the pressure reading in the altimeter setting window noted. B 5.5.7.4.8.a.1 J13 AIR,RTC Pilots are encouraged to turn on the aircraft rotating beacon A. just prior to taxi. B. anytime they are in the cockpit. C. anytime an engine is in operation. C 5.5.7.4.9.a.1 J14 ALL When in the vicinity of a VOR which is being used for navigation on VFR flights, it is important to A. make 90° left and right turns to scan for other traffic. B. exercise sustained vigilance to avoid aircraft that may be converging on the VOR from other directions. C. pass the VOR on the right side of the radial to allow room for aircraft flying in the opposite direction on the same radial. B 5.5.7.5.0.a.1 J27 ALL Choose the correct statement regarding wake turbulence. A. Vortex generation begins with the initiation of the takeoff roll. B. The primary hazard is loss of control because of induced roll. C. The greatest vortex strength is produced when the generating airplane is heavy, clean, and fast. B 5.5.7.5.1.a.1 J27 ALL During a takeoff made behind a departing large jet airplane, the pilot can minimize the hazard of wingtip vortices by A. being airborne prior to reaching the jet's flightpath until able to turn clear of its wake. B. maintaining extra speed on takeoff and climbout. C. extending the takeoff roll and not rotating until well beyond the jet's rotation point. A 5.5.7.5.2.a.1 J27 ALL Which procedure should you follow to avoid wake turbulence if a large jet crosses your course from left to right approximately 1 mile ahead and at your altitude? A. Make sure you are slightly above the path of the jet. B. Slow your airspeed to VA and maintain altitude and course. C. Make sure you are slightly below the path of the jet and perpendicular to the course. A 5.5.7.5.3.a.1 J27 ALL To avoid possible wake turbulence from a large jet aircraft that has just landed prior to your takeoff, at which point on the runway should you plan to become airborne? A. Past the point where the jet touched down. B. At the point where the jet touched down, or just prior to this point. C. Approximately 500 feet prior to the point where the jet touched down. A 5.5.7.5.4.a.1 J27 ALL When landing behind a large aircraft, which procedure should be followed for vortex avoidance? A. Stay above its final approach flightpath all the way to touchdown. B. Stay below and to one side of its final approach flightpath. C. Stay well below its final approach flightpath and land at least 2,000 feet behind. A 5.5.7.5.5.a.1 J27 ALL With respect to vortex circulation, which is true? A. Helicopters generate downwash turbulence, not vortex circulation. B. The vortex strength is greatest when the generating aircraft is flying fast. C. Vortex circulation generated by helicopters in forward flight trail behind in a manner similar to wingtip vortices generated by airplanes. C 5.5.7.5.6.a.1 J27 ALL Which is true with respect to vortex circulation? A. Helicopters generate downwash turbulence only, not vortex circulation. B. The vortex strength is greatest when the generating aircraft is heavy, clean, and slow. C. When vortex circulation sinks into ground effect, it tends to dissipate rapidly and offer little danger. B 5.5.7.5.7.a.1 J31 ALL As hyperventilation progresses a pilot can experience A. decreased breathing rate and depth. B. heightened awareness and feeling of well being. C. symptoms of suffocation and drowsiness. C 5.5.7.5.8.a.1 J31 ALL To scan properly for traffic, a pilot should A. continuously sweep vision field. B. concentrate on any peripheral movement detected. C. systematically focus on different segments of vision field for short intervals. C 5.5.7.5.9.a.1 J31 ALL Which is a common symptom of hyperventilation? A. Drowsiness. B. Decreased breathing rate. C. EuphoriA. sense of well-being. A 5.5.7.6.0.a.1 J31 ALL Which would most likely result in hyperventilation? A. Insufficient oxygen. B. Excessive carbon monoxide. C. Insufficient carbon dioxide. C 5.5.7.6.1.a.1 J31 ALL Hypoxia is the result of which of these conditions? A. Excessive oxygen in the bloodstream. B. Insufficient oxygen reaching the brain. C. Excessive carbon dioxide in the bloodstream. B 5.5.7.6.2.a.1 J53 ALL To overcome the symptoms of hyperventilation, a pilot should A. swallow or yawn. B. slow the breathing rate. C. increase the breathing rate. B 5.5.7.6.3.a.1 J56 ALL Which is true regarding the presence of alcohol within the human body? A. A small amount of alcohol increases vision acuity. B. An increase in altitude decreases the adverse effect of alcohol. C. Judgment and decision-making abilities can be adversely affected by even small amounts of alcohol. C 5.5.7.6.4.a.1 J58 ALL Hypoxia susceptibility due to inhalation of carbon monoxide increases as A. humidity decreases. B. altitude increases. C. oxygen demand increases. B 5.5.7.6.5.a.1 J62 AIR,RTC,LTA To best overcome the effects of spatial disorientation, a pilot should A. rely on body sensations. B. increase the breathing rate. C. rely on aircraft instrument indications. C 5.5.7.6.6.a.1 L52 AIR,RTC During preflight in cold weather, crankcase breather lines should receive special attention because they are susceptible to being clogged by A. congealed oil from the crankcase. B. moisture from the outside air which has frozen. C. ice from crankcase vapors that have condensed and subsequently frozen. C 5.5.7.6.7.a.1 L52 AIR,RTC Which is true regarding preheating an aircraft during cold weather operations? A. The cabin area as well as the engine should be preheated. B. The cabin area should not be preheated with portable heaters. C. Hot air should be blown directly at the engine through the air intakes. A